Download as docx, pdf, or txt
Download as docx, pdf, or txt
You are on page 1of 101

You need 107 correct answers out of 150 scored questions to pass.

1. Which of the following statements is an assumption of crisis theory?


a. Anger is a useful dynamic if consciously used
b. Insight is a prerequisite for change
c. Ability to cope with stress can be learned over time
d. Internal conflict prevents change

Rationale: While options A, B, and D are correct for other theoretical approaches,
option C is the only assumption that is appropriate to crisis theory.

2. The juvenile facilities in a city are clearly substandard. As a first attempt to


aid in correcting the problem, the social worker should:
a. Mobilize interest groups to explore and recommend alternatives
b. File a lawsuit to force the city to upgrade the facilities
c. Organize the parents of the juveniles to demand change
d. Upgrade prevention programs to keep more youths out of jail

Rationale: Key A is the correct answer in this situation since mobilizing interest
groups would be the best starting point to begin assessing the facilities and
establishing possible recommendations. B, C, and D might be done in the future,
but these steps would be determined based on the outcome of the interest groups,
which must be organized first.
3. In the process of referral, clients will MOST likely follow through to the
new agency if:
a. They have a clear understanding of what the new agency has to offer
b. The option to follow through is left up to the clients
c. They are informed of the consequences if they do not follow through
with the referral
d. They know that the agency is expecting them

Rationale: Unscored question. One of the basic roles performed by social workers
is that of brokering services, i.e., referring clients to appropriate community
resources. However, client follow through to actually connecting with the referral
agencies requires knowledge of the resources, which in turn requires agencies to be
skilled in the interpretation of these resources to clients. If the social worker is
unclear or vague about the services a referral agency offers option A, the client is
much less likely to follow through. Communicating a clear picture of an agency’s
services is prerequisite knowledge to strategies – B, C, and D – that might
encourage the client to request services.

4. According to the psychodynamic theory, when a client relates to the social


worker as though the social worker is someone from the client’s distant past,
the client is experiencing:
a. Transference
b. Projection
c. Conversion reaction
d. Introjection
Rationale: Basic recall question regarding the definition of transference. B, C, and
D are defense mechanisms.

5. In an initial interview a client appears suspicious of her surroundings and of


the social worker. She says that she wants to reveal certain facts but feels
that this would get friends in trouble. The appropriate response for the social
worker is to:
a. Encourage the client to talk about these things since she can trust the
social worker to maintain confidentiality
b. Tell the client that she understand how hard it is for her to feel safe in
this new situation and that she should reveal information when she
feels ready
c. Explain to the client how social workers maintain confidentiality and
show her how the files are locked to ensure privacy
d. Involve the client in a discussion of a less threatening subject

Rationale: Option A does not convey a message of acknowledgement of the


client’s concerns and appears to pressure the client into doing something she is not
prepared to do. Option C is not an appropriate response because a client would
never be shown how the files were locked, and such behavior does not necessarily
ensure trust in the social worker/client relationship. Option D avoids the
underlying issue of the client’s concerns and seems to divert the client away from
the client’s fears rather than addressing them. Therefore, B is the correct answer.

6. When critiquing a research article in a professional journal, a social worker


should minimally be able to:
a. Project the applicability of the findings to another group
b. Incorporate the results into policy planning at his or her own agency
c. Replicate the study in order to determine its reliability
d. Evaluate the appropriateness of the research design

Rationale: Unscored question. Social work’s emphasis on evidence-based practice


calls for social workers to use the best information available to inform practice
decisions. It is the responsibility of each social worker to make a judgment about
the quality of the research reported in the literature and its applicability to the
clients being served. The most fundamental skill, then, is to evaluate the design of
the research being reviewed, Option D. Only if the design is sound would the
social worker then be justified in applying the results to any specific client or client
group (A). Certainly a social worker would not invest time and resources in a
replication study (C) or use it as a basis for informing agency policy (B) unless
there was confidence that the design of the original research is sound.

7. A competent college student has been waking very early, has lost interest in
eating and socializing, and is not certain about her continued ability to
handle her studies. These symptoms MOST likely indicate:
a. Depressive disorder
b. Anxiety disorder
c. Anorexia nervosa
d. Insomnia

Rationale: It is the combination of these symptoms that make (A) the correct
response. Option B is incorrect because there are no symptoms indicating the
likelihood of anxiety disorder, whose predominant feature is worry. Option C is
incorrect because the decline in eating in anorexia stems not from loss of interest
but from a fear of gaining weight or becoming fat. Option D has only on
representative symptom in the stem (waking early), which makes this answer
incorrect.

8. A 45 year old client has made frequent references to death and violence. The
MOST accurate indicator of impending harm to self or others is:
a. The client’s continuous catharsis of extreme hostility
b. A history of violent behavior
c. Expression of a preferred method or plan
d. Excessive crying

Rationale: The more specific a client’s plan for suicide or violent behavior, the
greater the likelihood for carrying out the plan, making Option C the correct
answer. Catharsis of extreme hostility (A), history of violent behavior (B), and
excessive crying (D) are behaviors that, if taken independently, do not
automatically indicate the client will harm self or others.

9. When services are provided to a patient in the hospital, the social worker
makes regular entries in the patient’s medical record PRIMARILY to:
a. Demonstrate to the administration that the social service department is
providing adequate services
b. Protect the hospital should a legal suit arise
c. Facilitate communication among staff members regarding the care
that is being provided to the patient
d. Provide documentation for accountability purposes
Rationale: Option C is the correct answer. Documentation in the patient’s medical
record is primarily to communicate information regarding the care of the patient, as
the patient’s treatment relies on the social worker as the facilitator of the treatment
team. Options, A, B, and D are not the PRIMARY reason for documentation in the
medical record.

10.The concept of secondary gain in the psychodynamic approach is MOST


congruent with the social learning theory concept of:
a. Punishment
b. Extinction
c. Discrimination learning
d. Reinforcement

Rationale: Unscored question. The concept of secondary gain from the


psychodynamic approach refers to the benefit a person derives from an experience
in connection with a physical or mental health problem (e.g., getting sympathy
from friends and family) that reinforces the problem. In social learning theory, the
concept of reinforcement is congruent with secondary gain (D) because that
concept, too, is concerned with actions that support continuing a specific behavior.
The social learning concepts of punishment (A), extinction (B), and discrimination
learning (C) are not congruent with secondary gain.

11.A pregnant 17 year old has been referred to a school social worker. The
client is upset, confused, and uncertain about her options. After several
appointments with the client, the social worker recognizes that her own
values may be interfering with her relationship with the client. The social
worker’s MOST appropriate action is to:
a. Discuss the value dilemma with her supervisor
b. Refer the client to another social worker
c. Refer the client to a family planning agency for services
d. Arrange a family therapy session to discuss the pregnancy

Rationale: It is the social worker’s responsibility to identify how personal values


may interfere with the therapeutic relationship, and to seek consultation around
these issues (A). Referring the client to another social worker (B) or a family
planning agency (C) may be appropriate later, but the social worker must clarify
the impact personal values are having on the situation. The need for a family
therapy session (D) is not indicated.

12.Which of the following tasks is appropriate for a social worker utilizing a


strategic approach to family therapy?
a. Designing an intervention for each specific problem
b. Limiting interventions to the immediate family
c. Assuming a nondirective role with the family
d. Completing an in-depth family history

Rationale: Unscored question. The key to strategic family therapy is design


interventions to resolve specific issues, thus (A) is the correct answer.
Interventions may involve others beyond the immediate family, so (B) is incorrect.
The social worker may assume a somewhat directive role as the family seeks
strategies to problem-solving, so (C) is incorrect. Option D, completing a family
history, is not part of strategic family therapy as the history is not important in this
approach.

13.Which statement BEST describes resistance in the beginning stages of


treatment, according to psychosocial theory?
a. The client anticipates criticism from the social worker following self-
disclosure
b. The client minimizes problems and claim that the presenting problem
has been resolved
c. The client misinterprets statements made by the social worker
d. The client is dependent on the social worker for advice in dealing with
problems

Rationale: Psychosocial theory describes the various stages, life, tasks, and
challenges that every person experiences throughout the life cycle. The correct
answer B best describes resistance. The other answers are often impediments to
successful practice, but are not indicators of resistance.

14.A client has met weekly with a social worker for five weeks. The
relationship with the social worker has been positive. During the sixth visit,
the client becomes angry and challenging when the social worker makes a
clarifying comment. The social worker’s BEST response to the client’s
reaction is to explore:
a. The client’s understanding of the social worker’s statement
b. The client’s resistance to change
c. Limits for the client’s behavior
d. The client’s use of defense mechanisms

Rationale: This situation does not provide an example of a client’s resistance to


change, making Option B unacceptable. Exploring the limits of the client’s
behavior does not address the communication issue in this scenario, (C). Option A
is a better response than Option D, as it is possible that there was possibly a
misunderstanding of the social worker’s statement and not necessarily a defense
mechanism being used.

15.A husband has been committed to a psychiatric hospital as a result of violent


rages and assaultive behaviors directed toward his wife. In speaking with the
social worker, the husband blames his wife and threatens to harm her. Later
he retracts his threat and asks the social worker not to mention it to anyone.
The social worker should FIRST:
a. Notify the police
b. Document the threat and notify the treatment team
c. Request psychological testing to assess degree of danger
d. Explore the client’s ambivalence and develop a treatment plan

Rationale: The item addresses the primary duty of a social worker to ensure the
safety of the client and others. In view of the client’s recent history of violent
behavior, the social worker should be concerned about the potential for harm. All
of the options are actions the social worker might take in this situation; however
because the client is still in the hospital and the wife is not in immediate danger,
the FIRST action to take is B, documentation of the threat and notification of the
treatment team and the intended victim.
16.A social history with a 52 year old female admitted following a suicide
attempt reveals that she had been sexually abused for 6 months by a relative
when she was 12. In completing the social history, the social worker should
NEXT:
a. Take a complete sexual history
b. Obtain confirmation of this abuse from other family members
c. Determine how the abuse contributes to the client’s presenting
problems
d. Acknowledge but give little weight to the information since it is not
directly related to the immediate problem

Rationale: A and B may be helpful to the ongoing therapy, but neither one would
be the next step in this case. D is clearly the wrong answer because the social
worker has no idea whether the sexual abuse is important until it is understood how
it contributes to the presenting problem. Therefore, C, is what the social worker
should do next since it is important to understand how the abuse may have
contributed to this suicide attempt.

17.If a member of a therapy group breaches the confidentiality of another group


member ,the social worker should:
a. Have a joint interview with the two clients involved
b. Have an individual session with the client how has been betrayed
c. Terminate the client who breached confidentiality
d. Discuss the breach of confidentiality in the group

Rationale: The breach of confidentiality is a group issue because all of the


members of the group have agreed to maintain confidentiality of what is discussed
in the group. As such, the breach of confidentiality should be discussed in the
group. Option D is correct.

18.An elderly client is blind and confined to bed. Assistance with daily living
skills by a neighbor has allowed the client to remain at home. However, the
social worker is suspicious that the neighbor is taking the client’s money and
providing less than adequate care. The client wishes to remain at home,
continuing the arrangement with the neighbor. The home health staff is
concerned that interference in the situation will damage rapport with the
client. The social worker’s FIRST responsibility is to:
a. Discuss the situation with the agency’s lawyer
b. Report to adult protective services
c. Confront the neighbor about the suspicions
d. Work with the client to accept alternatives

Rationale: Although all options are actions the social worker might take, the
FIRST action is to report the situation to adult protective services (B). This is
based on the social worker’s suspicion that the client is being financially exploited.

19.An 18 year old single mother has a 20 month old child placed in foster care
for six months because of neglect. The mother has followed her reunification
plan, and the foster care plan is to return the child to the home. The foster
care social worker learns two weeks before the hearing that the mother is
several months pregnant. The worker’s BEST option is to request that the:
a. Mother gain legal and physical custody and close the case
b. Child remain in foster care until after the birth of the baby
c. Physical custody be returned to the mother and the family situation be
monitored by the agency for six months
d. Court date be postponed until the baby is born

Rationale: The mother in this vignette has complied with the reunification plan, but
is clearly in need of further monitoring because of the pregnancy. Restoring
custody and closing the case (A), is requesting that the child remain in foster care
until after the baby is born (B) and requesting that the court date be postponed until
the baby is born (D) would all be inappropriate and unethical. Best practice would
be to work toward custody while continuing to monitor the family situation to
determine whether the mother needs additional support (C).

20.A 67 year old who has recently retired seeks therapy with a social worker
because of confusion, irritability, and inability to concentrate. The social
worker should FIRST:
a. Refer the client for a comprehensive physical exam
b. Assign the client to a support group for recent retirees
c. Assure the client these symptoms are common following a major life
change
d. Explore adequacy of plans made for retirement

Rationale: (A) is the correct answer even though it may be important to accomplish
all of the other options at some point. It is always important when working with
any client to refer for a medical examination if there is any concern medically.
21.An adult client with no strong family ties is preparing for discharge from a
psychiatric hospital. Which of the following social work activities will
BEST contribute to the client’s successful reentry into the community?
a. Helping the client locate appropriate housing
b. Ensuring that the client has achieved maximum hospital benefit
c. Helping the client to apply for financial assistance
d. Collaborating with community agencies

Rationale: Because the client has little support from the family, it will be important
for the social worker to identify agencies which can be helpful following
discharge. The broader the base of support, the better the chance is for a successful
readjustment to the community.

22.The parents of children placed in foster care have made no progress after
extended casework services in improving the behavior that resulted in the
children being removed from their home. The child services agency staff has
determined that procedures to terminate parental rights will begin in the next
few months. The foster care worker’s NEXT step should be to:
a. Increase efforts with the parents to modify the parents’ behaviors
b. Begin to prepare the children for the transition of having parental
rights terminated
c. Ensure that the physical and mental health histories of the children’s
maternal and paternal families are available
d. Request that a legal guardian for the children be appointed

Rationale: Option B is correct. Once a decision has been made to terminate


parental rights, the next step for the foster care social worker should be to begin
working with the children on how this decision with affect them. Option A is
incorrect because the focus no longer is on addressing the parents’ behaviors.
Option C is not necessary for terminating parental rights. Option D may occur at
some point, but would not be the next step in this situation.

23.A social worker in a neonatal intensive care unit is approached by a hospital


volunteer asking which infants may be available for adoption. The social
worker’s BEST response is to:
a. Take the volunteer to view infants being placed for adoption
b. Refer the volunteer to a lawyer representing the parents seeking to
place an infant
c. Introduce the volunteer to parents seeking to place an infant in an
adoptive home
d. Decline to share such information with the volunteer

Rationale: Unscored question. The social worker should not share this information
with the hospital volunteer as this would be a violation of confidentiality.
Therefore, D is the correct answer. A, B, and C would all be violations of
confidentiality.

24.A social worker is consulting with the parents of a young adult who has been
recently diagnosed as having bipolar disorder. The social worker should
FIRST:
a. Obtain a family history of the client
b. Identify programs available for the client
c. Provide information about the disorder
d. Develop ways to maintain the client’s independence

Rationale: This question requires the test-taker to know when to use education as
part of the intervention process. Given that the child’s diagnosis is recent, it is
important that the parents have a full understanding about the child’s disorder (C).
It should not be necessary to take a family history (A) since the child has already
been diagnosed; identifying programs available to the client (B) and working on
independence issues (D) may be appropriate at some point, but are not the FIRST
actions the social worker should take.

25.A DSM-IV-TR diagnosis of major depression, single episode, is indicated


by all of the following symptoms EXCEPT:
a. Flat affect
b. Feelings of hopelessness
c. Hallucination and hearing voices
d. Suicidal gestures

Rationale: A, B, and D are three symptoms of major depressive disorder. Answer


C is not a part of a major depressive diagnosis because those symptoms are
characteristics of psychosis.

26.Which of the following techniques can assist a multi-problem client during


the intake process?
a. Free association
b. Interpretation of dreams
c. Partialization
d. Reflective consideration of dynamic content
Rationale: A client who presents with multiple problems often needs assistance in
identifying manageable entities to avoid feeling overwhelmed. As such, the correct
response is C.

27.Which of the following treatment paradigms uses the technique of “time


out”?
a. Social learning
b. Psychodynamic
c. Humanistic
d. Systems

Rationale: Option A is the only treatment paradigm that uses ‘time out’.

28.An 80 year old client has a double below the knee amputation which
requires weekly medical attention. The client is living with her son but
wants to relocate to an area with limited support services despite family
wishes. She has requested assistance from the hospital social worker. The
hospital social worker should FIRST:
a. Consult with the medical team to determine the client’s medical needs
b. Meet with the family to explore their concerns
c. Develop a plan with the patient to facilitate the move
d. Provide the client with information obtained from agencies in the
relocation area

Rationale: The stem clearly indicates that the client has medical needs that require
regular and ongoing medical care. The social worker has a responsibility to ensure
the client’s well-being, which means that the social worker’s FIRST step to
addressing the client’s wish to relocate must focus on determining the client’s
medical needs. Option A is correct.

29.A social worker makes home visits to a young mother with her first baby.
On the first visit, the 4 month old baby smiles, laughs, and responds to the
worker’s approaches. During a visit when the child is 8 months old the baby
turns way when the worker approaches, looks distressed, and begins to cry.
The social worker should interpret this change of behaviors as MOST likely
due to:
a. Conditioned fear response
b. Developmentally normal stranger anxiety
c. Child abuse
d. Normal temperamental irritability

Rationale: Unscored question. Option B is the correct answer because eight months
is the approximate age when children normally experience stranger anxiety,
according to attachment theory.

30.In facilitation a hospital support group for bereaved parents, a social worker
can discuss a personal experience with the death of a child to:
a. Create empathy with the participants
b. Receive help from the group
c. Create a sense of shared experience
d. Establish a role as expert on grief issues
Rationale: Unscored question. A social worker’s sharing of personal information
should occur only with the goal of serving a therapeutic purpose for the client.
Option C is the correct response.

31.A client reacts to criticism with rage, is interpersonally explosive, has a


grandiose sense of self, is preoccupied with thoughts of unlimited success,
and has a sense of entitlement. This meets the criteria for which of the
following DSM personality disorder diagnoses?
a. Paranoid
b. Antisocial
c. Narcissistic
d. Histrionic

Rationale: This question is a recall item that requires the test-taker to identify
symptoms associated with narcissistic personality disorder.

32.A hospital social worker encounters a family that has just experienced the
death of an infant due to sudden infant death syndrome (SIDS). The LEAST
appropriate social work intervention with the family until would be to:
a. Offer sympathy for the family’s loss
b. Help family members share their grief
c. Provide information on SIDS
d. Make funeral arrangements

Rationale: It would not be appropriate for a hospital social worker to make funeral
arrangements for a family so D would be correct. All of the other answers are
appropriate actions for a hospital social worker to take.
33.Which of the following social work activities would be considered a
PRIMARY prevention effort to address the problem of child abuse?
a. Helping design legislation which protects the rights of children
b. Providing shelters for children who have been abused
c. Conducting treatment groups for abusive parents
d. Delivering crisis intervention in home services for abusive families

Rationale: Helping design legislation which protects the rights of children (A)
would be a PRIMARY prevention effort, because if better legislation exists, then
children’s rights would be protected and child abuse would not be as prevalent. B,
C, and D would all be reactive efforts in response to child abuse which would have
already occurred.

34.A client has been evicted and is in need of immediate housing. The client’s
social worker owns an apartment building and has an available unit. The
social worker should:
a. Allow the client to temporarily rent the unit the social worker owns at
a reduced rate
b. Refer the client to several apartments, including the one owned by the
social worker
c. Refer the client to apartments owned by anyone not personally known
to the social worker
d. Refer the client to an apartment owned by a close friend of the social
worker who will give the client a reduced rate
Rationale: This question addresses the social worker’s obligation to avoid dual
relationships. The stem does not say that the client has no housing options
available other than the social worker’s apartment building. Referring the client to
apartments owned by others (C) is the answer that best avoids the dual
relationship. Including the social worker on the list of referrals (B) amounts to a
bending of ethics, referring to a friend’s apartment building (D) compounds dual
relationships, and renting to the client at a reduced rate (A) is a clear ethical
violation.

35.A 17 year old mother of an infant wants to return to high school to complete
her senior year with her class. The principal advises the student remain at
home until her child is older and informs her that evening classes are
available. The student seeks the services of the school social worker. The
social worker should:
a. Assist the student in understanding the principal’s recommendation
b. Meet with the principal and student to advocate on the student’s
behalf
c. Report the matter to the school board
d. Explore with the student alternative educational opportunities

Rationale: The social worker is being asked to assist the student in making plans to
complete her senior year of high school with the rest of her class. Options A and D
do not facilitate the student’s self-determination in reaching her goal. The social
worker might do option (C) but only after attempting to advocate on the client’s
behalf directly with the principal first (B).
36.A 15 year old girl has been sent home from school on three occasions for
drinking alcohol. Each time, the mother becomes angry and threatens to
punish the girl but is convinced by the father to give her another chance.
Which of the following terms BEST characterizes the father’s behavior in
this situation?
a. Triangulating
b. Enabling
c. Advocating
d. Manipulating

Rationale: Option B is the correct answer because enabling in this case is


facilitating dysfunctional behavior. D is not the right answer because there is no
discussion of any attempts to manipulate anyone in the information provided. A is
incorrect, because triangulation is the attempt by two individuals to collude against
another and this is not described in the stem. C is not the answer because
advocating is championing the rights of a client system.

37.A classroom teacher refers a 7 year old child to a school social worker
because the child consistently comes to school tired, unclean, and unable to
concentrate on lessons. The social worker’s FIRST action should be to:
a. Make a referral to child protective services
b. Refer the child for academic testing
c. Meet with the parents and the child
d. Refer the child to the school nurse

Rationale: This questions asks about the social worker’s FIRST action – it is
important to keep these qualifying words in mind when answering test questions.
In this case, an assessment of the family (C) must take place before any other
actions are taken. Only after this assessment occurs can the social worker identify
the most effective responses to this issue.

38.The phase of the helping process which leads to subsequent implementation


of interventions is:
a. Exploration and summation
b. Exploration, assessment, and planning
c. Initiation of establishing rapport
d. Clarification and analysis

Rationale: This is a basic recall item on the stage of the helping process.

39.A social worker has been treating a client for 2 months in an agency setting
and is experiencing intense sexual attraction toward the client. The social
worker’s BEST response is to:
a. Consult with an agency supervisor about the case
b. Discuss the attraction with the client
c. Transfer the client to another agency worker
d. Discuss the case with colleagues for feedback

Rationale: This question is focused on a fundamental principle of social work


values and ethics. Social workers experiencing inappropriate feelings towards a
client must seek consultation (A).

40.A child can walk well but still runs with an awkward gait. She enjoys
playing and pushing large objects such as carts and wagons. She plays with
other children, but is not able to play with them in a cooperative way. Her
vocabulary is about 25 words and she is able to put two to three words
together to express an idea. This child would be considered to be at the
developmental age of:
a. 9 months
b. 12 months
c. 2 years
d. 4 years

Rationale: Option C is the correct answer because this is the approximate age when
children reach these developmental milestones.

41.A 40 year old patient with a long history of substance abuse has been
diagnosed with AIDS in the advanced stage and is being considered for
discharge from a treatment facility. The patient’s elderly parents have
expressed willingness to have the patient return to live with them. In
formulating discharge plans, the social worker should FIRST evaluate which
of the following factors?
a. Availability and access to outpatient health services
b. The family’s ability to provide care
c. Patient’s likelihood of involvement in substance abuse
d. Availability of financial and vocational resources in the community

Rationale: In this question, the primary focus should be on the offer for care, and
the social worker’s first obligation is to evaluate the family’s potential to provide
this care (B). The availability of outpatient services (A) and the availability of
community resources (D) are important, but not as urgent as the evaluation of
primary care. The likelihood of substance abuse (C) would be a consideration in
any setting, but not the social worker’s FIRST area of evaluation in this scenario.

42.The PRIMARY source of information for a social history on a client


diagnosed with middle stage Alzheimer’s disease is the:
a. Individual client
b. Client’s family
c. Referral source
d. Attending physician

Rationale: Unscored question. There are seven stages of Alzheimer’s disease.


Between the third and fourth stages, family members tend to be the best source of
identifying dementia, therefore, the correct answer is B.

43.A social work administrator is designing an after school tutorial program for
school referred students with learning disabilities. On the administrator’s
staff are a psychologist and classroom teacher. Which additional
professional should be added to the staff to ensure a comprehensive
assessment?
a. Recreational therapist
b. School nurse
c. Reading specialist
d. Special education teacher

Rationale: Unscored question. Recreational therapists (A) would consider physical


needs more than learning. A school nurse (B) would not be trained to conduct
assessments for learning disabilities. A reading specialist (C) would consider only
one element of learning, where a special education teacher would take into
consideration many types of learning disabilities that affect more than just reading.
D is the correct answer.

44.Common indicators of sexual, physical, and emotional abuse include all of


the following features EXCEPT:
a. Violence against younger children
b. Sexualized behavior
c. Isolation
d. Stuttering

Rationale: Stuttering is not a common indicator of child abuse, therefore, D is the


correct answer. Violence against younger children (A), sexualized behavior (B),
and isolation (C) mentioned in the question are indicative of sexual, physical, and
emotional abuse.

45.The concept of gender roles is BEST defined as:


a. Attitudes and behaviors attached to individuals because of their sex
b. An individual’s sexual orientation and/or preference
c. Biological characteristics that influence an individual’s behavior
d. Interplay of biological and sociocultural influences

Rationale: Option A is correct. This is a recall item requiring knowledge of the


definition of gender roles.

46.A client frequently arrives late for appointments with her social worker and
sometimes fails to call in advance to cancel. When confronted, she
minimizes the problem and accuses the social worker of having
unreasonable expectations. The social worker’s MOST appropriate response
is to:
a. Ask the client to sign an attendance agreement and pay for missed
appointments
b. Encourage the client to talk about her feelings and perceptions
c. Transfer the client to another therapist
d. Offer the client a different appointment time

Rationale: Requires test-taker to understand the ways in which the social worker-
client relationship may be used as an intervention tool. Because the client has not
yet established a consistent pattern, and because the client reacts in the ways she
does when confronted about the missed appointments, the social worker’s MOST
appropriate response at this juncture would be to further explore the client’s
perspective (B). Requiring an attendance agreement (A) does not get at the root
causes of the behavior, transferring the client (C) and offering a different
appointment time (D) are both actions that avoid the cause entirely. Either C or D
could take place, but it is possible that the client’s behavior would not change
because of those actions.

47.A Latino child has been referred to a mental health clinic because of
disruptive behavior at school. The mother speaks limited English, is
confused, and does not understand the referral or the school’s concerns. In
the referral to a Spanish speaking social worker, the child’s male teacher
notes that the mother will not communicate with him and seems
uninterested. To facilitate an assessment, the social worker’s NEXT step is
to:
a. Arrange to go with the mother for a school conference
b. Observe the child in a classroom setting
c. Refer the child for a psychological evaluation
d. Schedule an appointment for a family assessment

Rationale: Option A is the first response in order for the social worker to determine
the mother’s level of comprehension during the conference. Once it has been
determined that the mother is aware of the nature of the referral and that
communication has been established, the social worker might at that time observe
the child (B), conduct a family assessment (D), and make a referral (C).

48.The purpose of feedback in a social work interview is to:


a. Allow the social worker to confront distortions in the client’s thinking
b. Summarize the main points of the interview
c. Let the client know the social worker understands the issue
d. Give the social worker the opportunity to self-disclose for the benefit
of the client

Rationale: This is a basic recall item that requires an understanding of the use of
feedback during an interview. Option A more accurately describes confrontation,
while option B is an example of reflection and clarification. Self-disclosure (D) is
not usually a part of providing feedback.

49.A client presents with numerous symptoms of depression. Her spouse of 22


years died recently. She is not sleeping or eating well and cries often. An
additional symptom which would lead to a diagnosis of major depressive
disorder rather than uncomplicated bereavement would be:
a. Withdrawal from close friends
b. A sense of hopelessness
c. Obsessive thoughts about the deceased
d. Talking about the deceased at every opportunity

Rationale: In diagnosis of major depressive disorder, hopelessness is a key


characteristic. In uncomplicated bereavement withdrawal from close friends,
obsessive thoughts of the deceased and talking about the deceased can certainly
occur but are not necessary for diagnosis of a major depressive disorder. Therefore,
B is the correct answer.

50.A 9 year old student was referred to the school social worker because of
aggressive behavior on the playground. In the first telephone contact with
the child’s mother, the social worker learns that the child has become more
aggressive at home since the mother’s remarriage 3 months ago. In this
situation, the social worker’s FIRST task should be to:
a. Help the teacher to design a behavior modification program
b. Intervene with the child on the playground
c. Meet with the parents for an assessment interview
d. Place the child in a peer support group

Rationale: In this scenario, the social worker has made phone contact with the
child’s mother and learns about a recent remarriage. The FIRST step now is to
complete an assessment with the parents (C) to determine an appropriate
intervention, which may or may not be A, B, or D.
51.A newly arrived immigrant family has a 14 year old daughter in junior high
school. The school is encouraging her to become involved in after school
activities. The girl tells the school social worker that she is not permitted
outdoors after school. The social worker should FIRST:
a. Encourage the parents to allow the student greater freedom
b. Tell the student that she can stay out during the daylight
c. Facilitate interaction with peers during school hours
d. Refer the student to support group

Rationale: Focuses on cultural differences, respect, and sensitivity to new


immigrants. Option C is the correct answer because it respects the family’s rules
about their daughter not being permitted outdoors after school, yet encourages the
14 year old to develop peer relationships.

52.During an interview with a client who has failed to meet the goals of a
contract, the client blames the social worker for the failure. This statement is
an example of:
a. Rationalization
b. Reaction formation
c. Compensation
d. Projection

Rationale: Option D is the correct answer because it is the only defense mechanism
listed that includes the blaming of another person, or attributing one’s own
unacceptable personality traits to another person. A is not possible because the
client is not presenting in logical terms why he or she feels the social worker is the
reasons for the contract failure. B is incorrect because the question does not
indicate that the client is behaving in a way that is opposite his or her values.
Option C is incorrect because the client is not trying to make up for imaginary or
real personal characteristics that may be considered undesirable.

53.A PRIMARY strategy for social workers interested in community building,


networking, and local level problem resolution is:
a. Interorganizational collaboration
b. Interagency consultation
c. An ad hoc agency committee utilization
d. Force field analysis employment

Rationale: Option B provides only information received in consultation. In dealing


with agencies only, the social worker may overlook available ad hoc or grassroots
organizations. Option C is a short-term solution that does not include community
building and problem resolution, and Option D offers only an assessment. Option
A includes assessment, strategies, goals, and long-term solutions, making it the
primary strategy.

54.Which type of interviewing question is MOST likely to generate client


defensiveness?
a. Open
b. Closed
c. Leading
d. Why

Rationale: People generally become defensive when they are asked why they
behave in a certain way or do certain things (D). The open questions (A) encourage
the client to talk more, closed questions (B) limit how the client can response, and
leading questions (C) cause clients to response in a certain way – but these do not
tend to generate defensiveness.

55.When interviewing a client whose speech patterns are different from the
social worker’s, the social worker’s PRIMARY consideration should be that:
a. Communication patterns vary due to economic conditions
b. Speech is influenced by culture and experience
c. Psychosocial conditions will cause variations in speech
d. Speech is influenced by group taboos and norms and may indicate
false beliefs

Rationale: Option B is correct. A and C are biased answers because they are
interpretations of stereotypes that those who are poor, uneducated, or diagnosed
with a mental disability will speak differently than the social worker. Option D
includes an inaccurate assumption that one can somehow relate speech to the
presence of ‘false’ beliefs.

56.A social worker meets with a couple who refuse to provide appropriate
medical care to their child with a life threatening illness because of their
religious beliefs. The child is receiving some care within the guidelines of
their church’s ministry but the child’s condition is deteriorating. How should
the social worker respond to this dilemma?
a. Take no action on the basis of client confidentiality
b. Not report the situation because the family’s right to self-
determination restricts the work’s actions
c. Report the situation to child protective services
d. Continue to strongly encourage the family to take a different action in
the best interests of the child

Rationale: Safety and well-being of the child is the primary focus of the social
worker, which makes reporting the situation to child protective services the only
option C. The limits of confidentiality are evident in this situation because of the
potential for harm towards the child (A). Not reporting the situation places the
social worker in a position of potential liability (B). Option D does not account for
the ethical and possible legal responsibilities faced by the social worker.

57.The biological parents’ rights have been terminated and the child is placed in
foster care. Some months later, the parents contact the foster care social
worker, requesting that their child be returned to their custody. The social
worker should:
a. Set up an agreement for changes needed before agreeing to reconsider
b. Advise the parents to talk to a lawyer about their request
c. Inform the parents that there is nothing the social worker can do
d. Request that the parents agree to a home study as preliminary to
reunification

Rationale: Termination of parental rights is a legal action. Although social workers


perform a role in helping the courts make the decision to sever parental rights and
subsequently monitor the care of the child, only the courts can reverse the legal
action. Thus the most appropriate assistance to biological parents wanting to
reverse a court action would be to help them find legal counsel (B). Any other
actions by the social worker, A and D, must first be requested by the court.
Indicating that the social worker could do nothing (C) would not be helpful to the
parents.

58.A mother and her 15 year old pregnant daughter seek consultation for the
daughter’s unplanned pregnancy. The mother wants the baby to be released
for adoption because she is in ill health and cannot help her daughter raise
the baby. The social worker’s FIRST response should be to:
a. Encourage the young woman’s mother to consider other options
b. Help formulate an adoption plan
c. Meet with the young woman alone to explore her feelings and wishes
d. Suggest that the mother participate in the treatment sessions

Rationale: Requires test-taker to apply the values of self-determination to social


work practice. Client self-determination in this scenario would be best served by a
private meeting with the young woman to explore her feelings about the pregnancy
(C). Trying to convince the mother to consider other options (A) would be
perceived as the social worker ‘taking a side’ in this situation at this point. Helping
to formulate an adoption plan (B) and involving the mother in treatment (D) may
be things that the social worker would do, but not as a FIRST response.

59.A couple consults a social worker because their child is disobedient. The
social worker is aware that the couple has little understanding of child
development and age appropriate behavior. To most effectively service this
family, the social worker should FIRST:
a. Let the parents know that their expectations are inappropriate
b. Discuss behavioral techniques to assist the parents in achieving their
goal
c. Acknowledge the parents’ concern and explore with them possible
methods of intervention
d. Recommend parenting classes for the parents to increase their
understanding of child behavior and parent effectiveness

Rationale: Correct response is C. As the FIRST task is to begin developing a


relationship with the parents. An effective way to do this is to acknowledge their
concern. The other actions A, B, and D may follow as the work begins.

60.A 10 year old child is referred to a school social worker for irritability, poor
concentration, and low self-esteem experienced during the past year. Which
diagnosis would be appropriate?
a. Oppositional defiant disorder
b. Communication disorder
c. Dysthymic disorder
d. Separation anxiety disorder

Rationale: Question requires test-takers to recall symptoms most strongly related to


dysthymic disorder in children.

61.A family with a 14 year old pregnant daughter has come for family therapy.
The parents are angry at the daughter and cannot agree with one another as
to the course of action they should take. The social worker should FIRST:
a. Establish trust with the daughter
b. Assess the family motivation for services
c. Evaluate the anger level of family members
d. Identify each family member’s view of the problem

Rationale: Item requires knowledge of a basic problem-solving model. Problem


definition and/or identification must occur first, before intervention strategies are
to be implemented, therefore, D is correct.

62.When selecting indicators of organizational performance, the MOST critical


factor a program evaluator should consider is the:
a. Ability of workers and managers to have some control over the
performance being evaluated
b. Length of time data related to the indicator has been collected
c. Interest of board members in the information to be collected
d. Potential public relations damage that could come from a negative
result on the indicator

Rationale: Although length of time the indicator has been collected (B) and the
interest of board members (C) may be significant, the MOST critical factor of
those listed is the control of the workers and managers over the performance being
evaluated (A). The outcome of a program will be different depending on this
variable. The potential public relations damage of a negative report (D) is not the
concern of the evaluator. Responsibility or concerns should be placed with the
administrator and the board of directors.

63.A hospital social worker sees a 55 year old Asian woman who hospitalized
for chronic, severe abdominal pain. Tests conducted do not reveal any
organic problems. History reveals that she is a refugee from Vietnam who
emigrated 15 years ago with her husband and children. The MOST
appropriate initial assessment of the client’s problem is that she is:
a. Being abused by her spouse
b. Somatizing emotional pain
c. Experience acculturation difficulties
d. Dealing with a midlife crisis

Rationale: Option B is the most appropriate answer following an initial assessment


because the absence of organic problems indicates that the pain is most likely to be
a result of somatization.

64.Which medication would MOST likely be used for treatment of depression?


a. Prolixin
b. Zoloft
c. Haldol
d. Zyprexa

Rationale: Question requires recall of the typical uses of various drugs.

65.An 8 year old was brought to a family service agency by her parents because
of enuresis and poor school performance. After the initial assessment, the
FIRST course of action for the social worker should be to:
a. Obtain a complete social history
b. Recommend psychological testing
c. Refer the child to a physician to rule out organic problems
d. Request school records
Rationale: Social workers must be able to recognize the presence of symptoms that
could have physical or neurological basis, and rule out these possibilities first. In
this question, the social worker should FIRST refer the child for a physical
examination (C) before approaching the issue as related to a behavioral factor. At
this point in the treatment, the social history should have already taken place (A),
medical causes need to be ruled out before referring for psychological testing (B),
and a request for school records may be appropriate later, but not until physical
causes have been investigate.

66.A social worker on an interdisciplinary team at a residential center for


children should make it clear to the residents that confidential disclosures
are:
a. Treated differently at times
b. Maintained between the resident and the social worker
c. Never shared with anyone
d. Shared with the treatment team

Rationale: Unscored question. Clients entering therapy with social workers must be
made aware of the limits of confidentiality when beginning services; this includes
children in residential treatment. This respects the client’s right to disclose or not
disclose certain information. The social worker cannot promise differential or
absolute confidentiality, particularly in a residential setting, so D is the correct
answer.

67.Which of the following statements is true of women who were victims of


incest during childhood?
a. They are less protective of their children from potential abuse
b. They disclose information about incestuous relationships openly in the
course of treatment
c. They all experience permanent damage as a result of abuse
d. They are more likely to experience depression and anxiety

Rationale: D is the correct answer. Although women may experience or


demonstrate those characteristics mentioned in A, B, and C, they are most likely to
experience depression and anxiety.

68.A client informs the social worker that while receiving marital therapy from
another social worker in the same agency, a business relationship developed
between the therapist and the client’s spouse. The social worker’s FIRST
course of action should be to:
a. Discuss the alleged ethical violation with the colleague
b. Assist the client in filing a complaint with the licensing board
c. Contact the client’s spouse to obtain more information
d. File a complaint with the agency

Rationale: Social work values and ethics question addresses the issues of how
social workers are expected to deal with colleagues who may be involved in dual
relationships. The alleged violation must be discussed with the social work
colleague first (A) before taking any additional steps.

69.A client has had diffuclty falling asleep for the past year. During an initial
session with a social worker in a medical clinic, the client expresses feelings
of hopelessness and a lack of desire for sexual relations. These symptoms
are MOST characteristic of:
a. Sexual aversion disorder
b. Depression
c. Anxiety
d. Adjustment disorder

Rationale: Recall question that requires knowledge of symptoms associated with


depression in adults.

70.During the second group therapy session, one member consistently criticizes
the group. The social worker facilitation the group should FIRST:
a. Encourage other group members to respond
b. Refocus on the group member’s contract with the group
c. Confront the group member
d. Ask the member to leave the group

Rationale: Unscored question. A is correct. By prompting group members to


respond to the statements, the social worker is beginning to teach group process.
The remaining three options would not be productive responses.

71.A physician tells the social worker that an elderly hospital patient will be
distressed by a nursing home placement. The physician instructs the social
worker to meet with the next of kin to arrange the placement without the
patient’s knowledge. The social worker should inform the physician that:
a. A range of discharge choices should be given to the patient
b. The patient has a right to be involved in the decision making process
c. Input should be sought from family members
d. A meeting with the patient and family needs to be arranged

Rationale: Social workers respect and promote the rights of clients to self-
determination and assist clients in identifying and clarifying goals. The social
worker has not assessed how the patient will react to a nursing home placement,
therefore, the correct answer is B.

72.Which of the following terms represents a combination of stereotyped


beliefs and negative attitudes which cause individuals to think about others
in a predetermined way?
a. Discrimination
b. Ethnocentrism
c. Racism
d. Prejudice

Rationale: Requires basic knowledge of the definition of prejudice, therefore,


Option D is correct. C – Racism is incorrect because there is not information in the
question related to race. B is incorrect because it does not indicate that it is a
general belief about the individual’s own culture. A is incorrect because it requires
actions instead of beliefs and attitudes.

73.A social worker, employed as part of a crisis response team with an urban
police department, is called to respond to an incident in which a man is
reported as behaving in a confused and erratic manner, including urinating in
a public place. The officer on the scene states that alcohol intoxication is
suspected. Upon arriving at the scene the social worker observes the man
removing his shirt and then sitting, slumped over on a park bench. The
social worker attempts to speak with the individual. The man mumbles a
word that sounds to the social worker like ‘diabetes’ but says little else. In
this situation, the social worker’s BEST response would be to:
a. Arrange for immediate transportation to a local psychiatric facility
b. Instruct the officer on the scene to administer a breath test to rule out
alcohol intoxication
c. Attempt to ask the client if he has his insulin with him
d. Have the man transported immediately to the nearest hospital
emergency room

Rationale: If there is any doubt regarding a client’s physical well-being, he should


be evaluated by a physician, so D is the correct answer. C is not correct because it
would be out of the social worker’s scope of practice. A would be inappropriate for
the social worker to do because this is a decision to be made by the officer and not
a directive to be given by a social worker. B would be a rash decision and based on
this man’s symptomatology, a medical explanation for this behavior must be ruled
out first.

74.When working with the families of dying patients, a social worker should
FIRST:
a. Encourage the family resolve past issues
b. Help the family express their feelings about the potential loss
c. Assist the family to understand the grief process
d. Work on the family denial system
Rationale: Unscored question. A social worker’s first job is to help the family
express their feelings, B is correct. Once the family has had an opportunity to
express their feelings for everyone to hear, the social worker would do C, then D,
and then A.

75.Treatment goals are MOST effective when they are:


a. Specific and measurable
b. General and flexible
c. Easily obtainable
d. Grounded in theory

Rationale: Definition of treatment goals states that they must be specific regarding
behaviors to be changed and stated so that the goal can be measured to see what
level of success has been achieved. A is the only correct answer.

76.A mother of three young children is hospitalized for an acute illness and will
remain in the hospital for approximately three days. The family is originally
from the Middle East. The father explains to the social worker that he
brought the children to the hospital so that they mother can continue her
duties and care for the children. The social worker should FIRST:
a. Inquire about the family’s culture and discuss options with the father
b. Refer the family to the local child welfare department
c. Inform the father of hospital policy which does not allow children to
spend the night
d. Review the mother’s medical condition to determine why she is not
able to care for the children
Rationale: Item addresses a social worker’s need to understand issues of diversity.
This item asks for the social worker’s FIRST action which should be to talk with
the family about their culture and how it relates to the mother and child care (A).

77.During a mandatory home visit to provide family preservation services, the


social worker finds the mother quite ill physically. The husband is present
playing games with the children while the mother attends to cooking and
other housekeeping chores. When the social worker expresses concern over
how much the mother is trying to do, the mother thanks her and states she
does not need any help. In response, the social worker should FIRST:
a. Explore the family’s management of household tasks
b. Offer to arrange homemaker services for the family
c. Encourage the mother to temporarily relinquish the chores
d. Provide a role model for the family by offering to assist the mother

Rationale: Item asks for the FIRST action. In order to understand why the mother
declined assistance, the social worker needs to assess the family patterns before
exploring solutions. Correct answer is A.

78.When conducting an assessment of the risk factors of child abuse for a child
of a different ethnic background, a social worker must FIRST:
a. Be aware of how personal cultural biases affect the social worker’s
ability to deal with issues of diversity
b. Realize that assessment models are not a substitute for clinical
judgment and experience
c. Use a strengths perspective rather than a deficit model to form a
partnership with the child’s family
d. Understand the need for appropriate eye contact, tone of voice, and
questioning techniques during the assessment

Rationale: Culturally competent social workers should develop an understanding of


their own personal and cultural values and beliefs as a first step in appreciating the
importance of multicultural identities in the lives of people, therefore, A is correct.

79.A clinical social worker in a seaside community notices that there are many
recently arrived Southeast Asians in the area, yet very few seek the services
of the local community mental health center. In an attempt to understand this
large underserved Asian population, the social worker needs to be aware of:
a. Indochinese cultural traditions
b. Special DSM-IV-TR protocols for this population
c. Traditional suspiciousness of western therapies
d. Limited comprehension of the English language

Rationale: In order for the social worker to be educated about this population, he or
she must have knowledge of their specific traditions.

80.After working with a client for two months, the social worker recognizes
that he has lost his objectivity and permitted himself to be pulled into the
client’s manipulative strategies. This is an example of:
a. Countertransference in the relationship
b. Transference in the relationship
c. Need for improved attending skills
d. A dual relationship

Rationale: Vignette requires the test-taker to recall the features of


countertransference.

81.A child is brought by parents to a social worker because of poor peer


relationships and a preoccupation with sexual matters. During the first
meeting, the parents report that the child has an unusually close relationship
with a grandmother. She is very protective of him, lets him stay up later at
night than his older siblings, and allows him to sleep with her. Given these
circumstances, the social worker should FIRST explore the possibility of:
a. Inconsistent parenting
b. Intergenerational conflicts
c. Scapegoating of the child
d. Incest

Rationale: There are several red flags in this scenario that would indicate possible
sexual abuse by the grandmother. A, B, and C are plausible answers, but the social
worker should FIRST look at the signals the child is displaying and the patterns of
behavior between the grandmother and the child. D is the correct response.

82.During an interview with a hospital social worker, a parent describes


multiple problems the family has experienced following the recent
diagnostic of the youngest child’s acute asthmatic condition. Given the
various problems faced by the family, the social worker should consider
which of the following actions?
a. Offer to work individually with the youngest child
b. Refer the family to a community agency
c. Request school reports for all children in the family
d. Conduct ongoing exploratory interviews with the parents

Rationale: Hospital social work tends to involve short-term efforts to help clients
deal with an acute medical problem. The most helpful action for a family
experiencing multiple problems that extend beyond the acute medical issue is to
connect them with a community agency that helps clients address multiple
problems and provide long-term services (B). It would be within the scope of
practice of a social worker at the community agency to which the client is referred
to begin collecting information for a full assessment of the family’s issues from the
parents (D) and the school (C) or to begin the long process of assisting the child in
adapting to the asthmatic condition (A).

83.A social worker who provides case consultation focuses PRIMARILY on:
a. Ethical considerations
b. The written case record
c. Specifics related to a client or situation
d. The credentials of the case manager

Rationale: Ethical consideration, Option A, is an element in every practice


situation. But it is not the primary focus when doing consultation. The written case
record, B, is only part of the information as there may be oral reports or other
assessments that are not included that may be relevant. It should not matter to the
social worker what the case manager’s credentials are when providing
consultation, (D). That information is irrelevant. Case consultation should be
directly focused on the specifics of that particular case or situation, C is correct.
84.A client seeks therapy with a social worker following a recent diagnosis of
multiple personality disorder and states that he is constantly in a state of
crisis. The social worker has no experience in this area and feels
uncomfortable working with the client. Social work values require that the
social worker:
a. Find a therapist skilled in the area to whom the client can be referred
b. Read literature on multiple personalities to better help the client
c. Help the client decrease the feelings of being in a state of crisis
d. Find an experienced social worker who would be willing to provide
occasional supervision

Rationale: A social worker must work in the best interests of the client and that
includes providing therapy only in those areas where there has been specific
training, therefore A is correct. Reading the literature does not train the worker to
an acceptable level of therapy (B), when the client is already in a crisis state.
Option C is part of the therapeutic process. Option D does not taken into account
the lack of training that the social worker has in that area and occasional
supervision does not address the social worker’s feelings of discomfort.

85.A client who is often fearful and suspicious has seen a social worker
intermittently for two years, and they have a strong relationship. The focus
of the interventions has been to promote client independence and personal
decision making. The client has an opportunity to move into a group home
that offers better social support than the current living arrangement. The
client is worried about making a change and reluctant to even discuss the
new housing option. The social worker asks the client to tour the home and
see what it is like before rejecting it. Because he trusts the social worker, the
client agrees. This is an example of:
a. Encouraging the client to use self-calming techniques to manage
anxiety
b. Compartmentalizing the client’s concerns to make them more
manageable
c. Empowering the client throughout the process of self-determination
d. Using the social worker/client relationship as an intervention tool

Rationale: Touring the home before making decision is not a self-calming


technique (A). The client did not provide many varying types of concerns which
would need to be compartmentalized, therefore, option B is not correct. Although
the client is using self-determination in order to make a decision, the issue focuses
more on the trust between the social worker and client than it does on
empowerment, so C is not the best answer. D is the correct response.

86.A social worker in a family service agency is asked by a client to contact her
spouse and let him know that she is in need of greater support from him. The
client explains that she feels her husband would listen to the social worker
better than he would listen to her. What should the social worker’s response
be to this request?
a. Explore the client’s dependency needs
b. Inform the client that this is inappropriate
c. Call the husband and present the client’s request
d. Discuss strategies the client can use to engage her spouse
Rationale: The social worker’s role is to empower clients to solve problems and
not to function as a broker in a relationship. In this scenario, the social worker’s
responsibility is to help the client find ways to express her feelings to her spouse
(D) - correct. Exploring the client’s dependency needs (A) represents a judgmental
approach; simply labeling the request as inappropriate does not directly help the
client (B). Calling the husband is an enabling behavior that may also be unethical,
depending on client consent issues (C).

87.During an intake interview with a social worker, a child cannot maintain an


appropriate conversation with the social worker, and engages in repetitive
use of words while talking. The child appears to be responding to internal
rather than external stimuli during the interview. The parents report that the
child has no physical problems but has been noticeably slower than their
other children in developing verbal and nonverbal communication skills.
According to the DSM-IV-TR, which disorder is the child MOST likely
experience?
a. Rett’s
b. Autism
c. Trichotillomania
d. Childhood disintegrative

Rationale: Requires knowledge of childhood disorders. The parents reported that


this child has been noticeably slower than their other children in developing verbal
and nonverbal communication skills, therefore, B is correct. Option A is
characteristic pattern of head growth deceleration, loss of previously acquired
purposeful hand skills and severe impairment in expressive and receptive language
development. Option C refers to recurrent pulling out of one’s hair that results in a
noticeable hair loss. Option D can only be diagnosed if symptoms are preceded by
at least two years of normal development; characterized by a clinically significant
loss in previously acquired skills.

88.A social worker is treating a client due to longstanding depression. In the


course of therapy, the social worker realizes the client has racist beliefs that
clash with the social worker’s value system. The social worker should:
a. Respectfully confront the client’s prejudiced beliefs
b. Educate the client about the negative effects of racism
c. Refer the client to another therapist who might have similar beliefs
d. Respect the client’s beliefs and focus on the presenting problem

Rationale: Option A and B involve the assumption that the social worker’s beliefs
take precedence over the therapy for depression. The two issues are not related so
the social worker should continue to focus on the presenting problem. It is not
necessary to refer the client to another therapist with similar beliefs (C) as the
client is in therapy for depression, not for issues involving racism. Therefore D is
correct.

89.A social worker at a prison meets weekly with a group of inmates who are
fathers. After ten weeks of group treatment the social worker instructs the
inmates to act out newly acquired parenting skills. This technique is known
as:
a. Role re-equilibrium
b. Role playing
c. Psychodrama
d. Behavior shaping

Rationale: By acting out learned roles, the fathers are rehearsing new behaviors
thus role playing (B) is correct. In psychodrama (C), clients play themselves in
stressful situations so they are more in touch with their feelings. Behavior shaping,
(D), would involve techniques such as reinforcement that are not primary to role
playing.

90.Due to the recent death of a parent, a hospital social worker is experience


difficulty concentrating and completing work related tasks. A colleague
reports to the social worker that patients are complaining about the social
worker’s irritability. The social worker should:
a. Schedule personal leave
b. Self-monitor reaction to patients
c. Appropriately disclose the loss to patients
d. Seek consultation with the supervisor

Rationale: Social workers experiencing difficulties providing services must


assume professional responsibility for their behavior and seek assistance, ideally a
supervisor (D). Scheduling personal leave (A) would not necessarily solve this
problem, self-monitoring (B) may be used in conjunction with consultation, but is
not likely to be a sufficient response on its own, self-disclosure (C) is sometimes
appropriate, but should not be used until the social worker has a better
understanding of his or her own reactions to the stressor.

91.In preparing service plans for a family of four children in foster care, the
social worker should recognize it is MOST important to:
a. State the exact reason the children were removed form their own
home
b. Discuss details of the cultural and ethnic background of the foster
parents
c. Develop separate goals for each sibling based on individual needs
d. Establish goals for the foster parents’ home

Rationale: Option C is correct. In order for the social worker to be effective in


providing services to the children, separate goals for each sibling based on
individual needs (C) must be developed. The other information, such as reasons for
removal (A), cultural and ethnic background of the children (B), and goals for the
foster parents’ homes (D) are important, however this information alone would not
sufficiently address the needs of the children in foster care. The development of
separate goals is clearly MOST important.

92.A social worker employed at a neighborhood youth center is meeting with a


17 year old. The youth raises personal concerns related to sexual identity and
homosexuality. How should the social worker respond to these concerns?
a. Refer the client to a therapist who specializes in treating sexual
struggles of adolescents
b. Provide therapy to the youth with the goal of resolving issues
surrounding homosexuality
c. Discuss with the youth accurate information concerning
homosexuality
d. Suggest a session with the client’s parents to discuss the concerns of
their child
Rationale: Unscored question. The youth is concerned about sexual identity, so the
first thing the social worker should do is provide information about homosexuality,
(C), to help the youth understand sexual orientation. The other options assume that
the presenting problem has clearly been identified.

93.A medical social worker is interviewing a recently widowed patient who was
hospitalized for treatment of severe malnutrition. The patient has responded
well to treatment and will be ready for discharge within a few days. The
patient has no immediate family and reports having been financially
exploited by acquaintances following the spouse’s death. In helping the
patient make discharge plans, the social worker should FIRST:
a. Refer the client to a bereavement support group
b. Ensure adequacy of the client’s financial resources
c. Seek legal assistance concerning the financial exploitation
d. Evaluate the client’s competence for self-care at home

Rationale: purpose of this interview is discharge planning which entails an


evaluation of the patient’s competence for self-care. Discharge planning interviews
involve examining the client’s support systems, and are an important element in
the development of the client’s post-discharge care plan. Therefore, option D is the
correct answer. Options A, B, and C have to do with social service needs, and
should only be addressed after the primary areas of concern have been
investigated.

94.A client discusses child care facilitates in another region, informing the
social worker that she plans to accept a job offer there. She is planning to
take her 4 year old child with her, although she understands that it would be
easier for her to remain in her present living arrangement with her parents
until the child is older. Given the client’s desire to move, the social worker
should:
a. Make arrangements for child care
b. Request immediate extended family meeting
c. Explore the pros and cons of the client’s plan
d. Ask the client not to take action for a period of time

Rationale: Because a client’s right to self-determination is important, the social


worker’s role is to assist the client in thinking through her decision to help the
client make best choice. Option C is correct. By asking the client not to take action
for a period of time (D) or by requesting an immediate extended family meeting
(B), the social worker is not respecting the client's right to make her own decisions.
Making child care arrangements (A) would be a premature action at this point.

95.A 12 year old girl has been referred to a social worker by her pediatrician
due to her sudden refusal to attend school. She complains of difficulty
breathing, pain in her chest and dizziness when she goes to school. Although
a medical evaluation is negative, she reports being afraid that she is dying.
The MOST appropriate diagnosis is:
a. Pervasive developmental disorder
b. School phobia
c. Separation anxiety disorder
d. Somatization disorder
Rationale: Option B is the most appropriate option since the refusal to attend
school has happened suddenly. Option A would be incorrect because there is
nothing in the question indicating that the 12 year old has a developmental disorder
which can be pervasive and long standing. Option C would not be correct given the
age of the girl. Option D would not be correct because the school refusal just
started and the question does not indicate that there has been a long history of
unsubstantiated physical complaints.

96.A young adult is in therapy with a social worker for depression following the
breakup of a long term relationship. The client’s mother contacts the social
worker and requests information on what is happening in the client’s
sessions. The social worker’s INITIAL response to this request would be to:
a. Arrange a family session so the mother can discuss her concerns
b. Give the mother general information about the client’s progress
c. Clarify with the mother the nature of confidentiality within the
therapeutic relationship
d. Explore with the mother her need to know what is discussed in
therapy

Rationale: Question addresses fundamental social work values and ethics around
confidentiality. Without client consent, no information can be shared (excepted in
limited circumstances). Arranging for a family session (A) would require the
consent of the client first, providing information to the mother (B) is a clear
violation of client confidentiality, exploring the mother’s reasons for wanting
information (D) inappropriately positions the mother as client. The only ethical
option is for the social worker to explain the basics of confidentiality in the social
worker-client relationship (C).
97.Before initiating a contract, social work consultants should:
a. Investigate the goals of the organization to ensure they can support
them
b. Investigate the solvency of the organization
c. Determine if the guidelines of past consultants have been followed
d. Examine personal values and conflicts that may arise

Rationale: A social worker would need to determine if the goals of the organization
are applicable to the information and type of expertise requested, therefore (A) is
correct. If not, then the organization would have to seek consultation from another
source. The solvency of the agency (B) should not have any bearing on whether
consultation takes place as the consultation itself may be related to this issues.
While it may be important to have an awareness of the guidelines that previous
consultants offered (C), this is not a primary consideration when initiating a
contract. Past guidelines would come later as part of the gathering of information
for the consultation itself. A social worker would need to know what the goals of
the organization are before examining any potential value conflicts (D).

98.A mother brings her teenage daughter to a social worker following a family
argument. The adolescent wants to feel that her feelings and ideas are heard
and respected by her parents. Her mother agrees that communication
between them needs to improve. In developing a treatment plan, the social
worker should write the objective as:
a. The client will increase her ability to talk with her parents by 50
percent
b. The client will show improved communication skills 80 percent of the
time
c. The client will communicate well with her parents in seven out of
seven trials
d. The client will report no more than one argument per week with her
parents

Rationale: Unscored question. An objective has a specific time frame as well as


measurable outcomes. Option D is the answer, as it is the only response which
includes both. Options A, B, and C contain outcomes which are neither specific
nor measurable.

99.A client at a family services agency discusses with the social worker recent
behavioral changes demonstrated by the client’s young child. The child has
become less talkative and appears to be withdrawn. In response to the social
worker’s questions, the client indicates that the changes first appeared after
the parent and child moved into the home of the client’s companion. The
social worker should:
a. Inform the client that the possibility of child abuse should be
considered
b. Determine if arrangements can be made for the child to stay with a
relative
c. Suggest that the child’s behavior is developmentally appropriate
d. Obtain more information about the environmental changes that have
occurred
Rationale: Question addresses the care that must be taken during assessment.
Although the social worker may have theories about underlying causes for
behavior, these theories should not be acted upon until more information is
obtained (D). Discussing possible child abuse (A) and exploring alternate living
arrangements (B) are, at the very least, premature. The child’s behavior is not
necessarily developmentally appropriate (C).

100. A physician refuses to inform a patient of a terminal illness and


hospital social worker is opposed to keeping this information from the
patient. The social worker should:
a. Inform the patient of the illness and prognosis
b. Discuss the concerns with the physician
c. Schedule a meeting of the team treating the patient
d. Report the physician to the jurisdiction’s medical licensing board

Rationale: The social worker should first try to resolve this ethical concern with the
physician (B). And while following agency protocol to address the matter. If this
does not resolve the issue, then Option C should be pursued.

101. A client uses clinical social work services to address issues of


emotional grief and abandonment. The client decides to stop treatment due
to loss of employment. The client declines to attend termination sessions
because of concerns about spending money while unemployed. The social
worker should:
a. Explain by phone why the client needs to complete the sessions face
to face
b. Offer termination sessions at a reduced fee to help the client discuss
events and gain a sense of closure
c. Encourage the client to come in and say goodbye and provide three
appropriate referrals
d. Have the client sign a form stating the client is terminating against
professional advice and is welcome to return

Rationale: Social workers should ensure that fees are fair, reasonable, and
commensurate with services and take into consideration clients’ ability to pay.
Therefore, offering this client a reduced fee would be appropriate as it addresses
the financial hardship that may have been caused by unemployment. Option B
would also allow the client further work on issues related to loss and abandonment.

102. A social worker at a mental health center overhears another social


worker agreeing to meet a recently terminated outpatient client later that
night to go dancing at a local bar. The social worker in this situation should
FIRST:
a. Meet with the coworker to voice concerns about the behavior with the
client
b. Contact the licensing board to receive clarification about the incident
c. Meet with the clinical supervisor to report the coworker’s behavior
d. Request a meeting with the supervisor and the coworker to address the
issue

Rationale: A social worker would discuss the matter with the coworker first in
order to alter the coworker of the possible ethical consequences, so A is correct.
The social worker would then meet with the supervisor to report the incident and
then possibly meet with both the supervisor and coworker, options C and D. The
final step if the behavior continues, would be to contact the licensing board (B).

103. In the final session of treatment with a social worker, a client brings
up new issues and expresses opposition to terminating. The social worker
should FIRST:
a. Explain the termination contract
b. Contract for an additional session
c. Discuss feelings about ending therapy
d. Explore new issues

Rationale: As indicated in the question, the client is having difficulty ending


treatment. The FIRST step for the social worker is to discuss how the client feels
about this transition, as this is obviously the client’s issue. Any other actions would
follow this discussion. Option C is correct.

104. A social worker receives a referral for hospice services for a patient
whose physical condition is rapidly deteriorating. The patient agrees to
hospice services only is she can be discharged to her adult daughter’s home.
The daughter feels she is unable to care for her mother at home. The social
worker should FIRST:
a. Ask for an ethics committee consultation
b. Encourage the patient to consider inpatient hospice
c. Schedule a patient care conference with the family
d. Meet with the daughter and explore her concerns
Rationale: Unscored question. Option A is not correct because this is not an ethical
issue. The daughter is probably overwhelmed at this point and may not understand
what would be involved. Option D is therefore the correct answer as it would be
helpful for the daughter to be able to express her concerns FIRST. If after
clarifying her concerns, if she is still not able to provide the care, the conference
should be scheduled (C). Option B may then be an outcome of option C.

105. In an intake session with a social worker, a recently married client


describes name calling, teasing, and yelling by her husband. She tearfully
asserts that it is her own fault and that she wants therapy in order to be a
better wife for him. The social worker should FIRST:
a. Educate the client about the dynamics of domestic abuse
b. Recommend martial therapy for the couple
c. Encourage the client to report her husband to the police
d. Discuss with the client the issue of dependency

Rationale: Question is asking about the FIRST task that the social worker should
take. While the other options are actions that may be appropriate, problem
definition must occur first, so option A is correct.

106. The initial symptoms of schizophrenia MOST often occur between


which of the following years of age?
a. 6 to 12
b. 13 to 16
c. 17 to 45
d. 46 to 60
Rationale: Research on schizophrenia indicates that it occurs MOST often between
17-45

107. A social worker meets with a client who has been treated with
antipsychotic medication for many years. The client reports uncontrollable
movements of the tongue, lips, and mouth. The social worker notices
repetitive movements of the client’s hands and legs which seem to be
involuntary. The social worker refers the client to the prescribing physician
to address possible symptoms of:
a. Akinesia
b. Dyskinesia
c. Akathisia
d. Dystonia

Rationale: Basic recall on knowledge of motor or movement disorders. Option A


refers to reduced or minimal motor movement. Option C refers to the inability to
sit down because the thought of doing so causes severe anxiety. Option D refers to
abnormal postures and dysfunction of involuntary muscle activities, resulting in
tics and spasms. Option B is correct – symptoms are those of tardive dyskinesia
(TD) which is a major side effect of prolonged use of antipsychotic medications.

108. A social worker is completing a social history on a 38 year old client


who is taking Lithium, prescribed by a psychiatrist. Based on this
information, what is the client’s MOST likely diagnosis?
a. Schizophrenia
b. Obsessive compulsive disorder
c. Bipolar disorder
d. Major depression

Rationale: Requires recall of how Lithium is used in treatment.

109. The PRIMARY purpose of obtaining a signed informed consent


before a voluntary interview is to:
a. Let the interviewee know that illegal activities will be reported to the
appropriate authorities
b. Advise interviewees that everything they say will be held in strictest
confidence
c. Establish a relationship and get the interviewee to feel free to speak
openly
d. Establish the specific content, purposes, and uses of the interview

Rationale: Best practice would be to obtain written informed consent to ensure that
the clients understand the purpose of the interview. Written consent also protects
both clients and social workers in cases of litigation, therefore, D is correct.

110. A client begins therapy with a social worker, requesting help in


clarifying feelings for a current sexual partner. After several sessions, the
social worker realizes that they client’s partner is a personal friend who has
often confided in the social worker about difficulties with this relationship.
The social worker should:
a. Continue working with the client without revealing the social
worker’s relationship with the client’s partner
b. Explain to the client the social worker’s relationship with the partner
and allow the client to decide whether or not to continue treatment
with the social worker
c. Tell the client about the social worker’s relationship with the partner
and refer the client to another therapist
d. Terminate the friendship with the client’s partner and continue
treating the client

Rationale: Option A creates conflict of interest for the social worker. Clients in
treatment should not be required to make the kinds of decisions referred to in
Option B, since this would create a conflict of interest for the client. Option D is
not a realistic expectation, therefore, C is correct.

111. During the initial phase of treatment, the role of the social worker can
BEST be clarified by:
a. Developing a contract to be followed
b. Defining the therapeutic modality to be employed
c. Discussing transference and countertransference issues
d. Determining the client’s expectations

Rationale: Initial phase, the primary focus is on the client’s expectations for the
treatment, so D is correct. The next step would be to develop a contract (A) based
on the client’s goals and then defining the treatment modality to be used (B).
Option C would only take placed during the treatment itself and only if
transference and/or countertransference became an issue.
112. Which of the following actions could BEST be used by the social
worker to convey acceptance to a client who is paranoid?
a. Use relaxation techniques
b. Maintain constant attention and direct eye contact
c. Offer assurance and gentle guidance
d. Verbally reframe what the client has said

Rationale: Option A may not help with a client who is paranoid as it does not
necessarily provide a trusting, accepting environment for the client. Option B may
have the client feeling even more uncomfortable with the direct eye contact.
Option D offers only clarification on the client’s statements but does nothing to
convey acceptance. Therefore, C is correct.

113. A social worker has been asked by the court to evaluate a family after
there have been five substantiated child abuse complaints. There are two
children under four years of age and the parent has been arrested three times
for driving while intoxicated during the last 12 months. The BEST
indication that the abuse may continue is:
a. Low economic status
b. Substance abuse by the parent
c. Multiple children in the family
d. The parent was abused as a child

Rationale: Requires test-takers to have knowledge about factors that contribute to


ongoing abuse. Parental substance abuse (B) has the strongest correlation. The
other options may contribute, but are not the ‘strongest indication that the abuse
may continue’.
114. The social development of a six year old child can be determined as
age appropriate when the medium of lay is PRIMARILY:
a. With opposite sex friends
b. In parallel play
c. With same sex friends
d. In mixed sex groups

Rationale: Stage of development, the primary medium of play is considered age-


appropriate when it is with the same sex (C), as sex and gender roles are beginning
to develop. A, B, and D are also age-appropriate, but are not PRIMARY medium
of play.

115. A social worker is assigned to assess an 8 year old child who is


exhibiting impulsive, aggressive, and argumentative behavior at home. The
child has daily tantrums, hits his parents when angry, and refuses to follow
any directions. This behavior has been ongoing for several years, and the
parents believe that it is getting progressively more severe. The school
reports minimal behavior problems, and the child is earning positive grades.
The MOST likely diagnosis for this child is:
a. Depression, childhood onset
b. Attention-deficit hyperactivity disorder
c. Bipolar disorder
d. Oppositional defiant disorder

Rationale: Oppositional defiant disorder is almost invariably present in a home


setting, but may be absent at school, thus D is correct. Childhood onset of
depression (A) and attention-deficit hyperactivity disorder (B) are more likely to
affect all settings, while bipolar disorders (C) does not reflect the consistent nature
of the behaviors.

116. To meet legal and ethical requirements, a supervisor responsible for


evaluating a social worker’s performance MUST base the evaluation on
which factor?
a. A detailed job description
b. Evidence of productivity
c. Effectiveness in service to clients
d. Work habits and professionalism

Rationale: The social worker’s job description defines the agency’s expectations
and standards, and serves as the basis for the social worker’s evaluation. Service
effectiveness is certainly a part of the evaluation, but the evaluation itself must be
based on the job description (A).

117. A social worker witnesses a colleague asking a client to purchase


toiletries at the discount store for the colleague, and giving the client money
for the products. The social worker confronts the colleague. The colleague
replies that the client goes to the store daily, and the client does not lose any
money so there is no problem with the situation. The social worker should:
a. Report this to the supervisor for more attention
b. Do nothing more since the issue was addressed
c. Talk to the client directly about the situation
d. File a complaint with the licensing board
Rationale: Social workers should first discuss unethical concerns with the
colleague in question when feasible. If this fails, as it does in the scenario above,
best practice would be to inform the supervisor for further exploration of the
situation.

118. A social worker is engaged in treatment with an adolescent client who


has been struggling with questions concerning his sexuality. The client has
determined that he is gay, but due to fears of rejection has chosen to keep his
sexual orientation private. The social worker needs to be aware that this
client, more than other adolescents, is at increased risk for:
a. Delinquent acting out
b. Sexually transmitted disease
c. Incomplete moral development
d. Paraphilia tendencies

Rationale: Because this client has been struggling with his sexuality, the risk of
acting out (A), far outweighs the risk for sexually transmitted diseases (B),
incomplete moral development (C), and paraphilia tendencies (D).

119. A social worker is studying the communication practices and values


of several other ethnic populations. To become proficient in multicultural
practice, it is essential that the social worker also:
a. Live and work in communities primarily populated by people from the
groups being studied
b. Become aware of personal cultural experience, biases, and values
c. Adopt the communication style of the members of the group currently
being studied
d. Undergo psychotherapy to address personal biases regarding other
cultures

Rationale: Self-awareness of the social worker’s own cultural background and


values (B) is a crucial component of competent practice, particularly in settings
that require multicultural practice. Living and working amongst other cultural
groups (A) may be effective, but it is not essential to becoming proficient –
likewise, with adopting a communication style consistent with member of the
group (C). There is nothing in the stem to indicate that psychotherapy is required at
this point (D).

120. A social worker has been providing case management services to a


client in a small agency where many of the clients known each other. The
social worker has heard through other clients that the client is unhappy with
the services the social worker is providing and thinks that the social worker
is mean. The social worker should:
a. Talk to the client about assertiveness skills and ask that future
concerns be discussed directly with the social worker
b. Tell the client about what has been said and share thoughts and
feelings about being called mean
c. Share with the client the rumors that have been heard and ask open-
ended questions about the nature of the problem
d. Discourage the other clients from gossiping and make an effort to be
kind to the client in the future
Rationale: First step in the problem solving process is to gather information. By
asking open-ended questions the social worker can better understand why the client
may have made these statements; therefore, correct answer is C. A, B, and D
reflect interventions that should not be used before the worker knows what the
problem is.

121. A social worker begins treatment with a new client who has just
suffered a significant loss. The social worker can BEST convey empathy by:
a. Expressing sympathy for the client
b. Using body language to communicate
c. Using reflective listening
d. Relating a personal experience of loss

Rationale: Empathy is BEST conveyed through demonstrations that the social


worker understands what the client is expressing – a technique central to reflective
listening (C). Sympathy (A) is not the same thing as empathy, using body language
(B) to communicate empathy is impractical, and self-disclosure (D) is not
necessarily an empathetic response.

122. A social worker at a mental health center is providing therapy to a


client who is court mandated to receive treatment. In writing case notes on
this client for the court, the social worker should make sure to include
information related to the client’s:
a. Understanding of the consequences for noncompliance
b. Degree of engagement in the treatment process
c. Progress toward specific service goals
d. Ongoing need for treatment services

Rationale: Unscored question. In a court mandated case situation, the courts would
need information to show that the client is working toward the goals specified in
the service mandate. It is not necessary for the courts to be aware of the other
options, as they do not relate to the courts. The information in the other options
would only be for the benefit of the social worker when deciding on a treatment
plan, thus C is correct.

123. The executive director of a social work agency is visited by a direct


service social worker who expresses various complaints about the agency.
The executive director should address this behavior by:
a. Initiating a series of meetings with staff to determine the validity of
the social worker’s complaints
b. Listening carefully by redirecting the social worker to the immediate
supervisor
c. Having the immediate supervisor point out to the social worker the
inappropriateness of such actions
d. Requesting a joint meeting with the executive director, the social
worker, and the social worker’s immediate supervisor

Rationale: B is correct. The executive director should listen to what the social
worker has to say, but also offer guidance to redirect the social worker to the
correct chain of authority, which is the social worker’s immediate supervisor.
Acting to initiate meetings with other staff (A) circumvents the immediate
supervisor and therefore, would undermine the supervisor’s authority and
administrative structure. Having the social worker’s supervisor point out the
worker’s inappropriateness (C) is an example of indirect communication from the
executive director and this is not the best way to handle the situation. A joint
meeting (D) may be called for in the future if the complaints merit it or are not
resolved, but there is nothing in the information given to indicate that this would be
the approach to take now.

124. An applicant at a social service agency requests help in locating a


larger apartment for the family. In the initial discussion with the social
worker, it becomes obvious that marital difficulties are also a major concern.
The social worker should FIRST:
a. Discuss the presenting problem
b. Request to see the spouse
c. Refer the client to marital counseling
d. Give the persona list of real estate brokers

Rationale: Requires test-taker to demonstrate the value of addressing the client’s


concerns first. The applicant in this scenario is requesting help with housing, and
that is what should be attended to first (A). Regardless of whether the social
worker feels that marriage difficulties are also an issue. Option D is a passive and
not very helpful way of responding to the presenting request. B and C do not
address the primary needs as expressed by the client.

125. A social worker at a mental health center has a client who has been on
antipsychotic medication for several years. The client has begun to have
uncontrollable shakiness and difficulty walking. The social worker makes a
referral to a psychiatrist for an evaluation. Since the client has been on the
medication for an extended period of time, the social worker is MOST likely
concerned about the onset of:
a. Epilepsy
b. Korsikoff’s disease
c. Parkinson’s disease
d. Tardive dyskinesia

Rationale: Tardive dyskinesia is associated with the use of antipsychotic


medications, and should be considered the MOST likely reason for the changed
behaviors (D). This is not to say that other physical conditions such a Parkinson’s
disease or epilepsy are impossible, but that the possibility of chemically-induced
influences needs to be considered first. Korsikoff’s disease (B) is associated with
prolonged alcohol abuse – something not indicated.

126. A social worker employed as a case manager at a community mental


health center has an adult client who is HIV positive. The social worker has
initiated a referral to a treatment group for HIV positive individuals. The
representative of the organization providing the group treatment services has
asked about the sexual orientation of the client, stating that the information
will assure a better fit between client need and services provided. In this
situation, the social worker should proceed by FIRST:
a. Releasing the information since it will be held in confidence between
service providers
b. Informing the person requesting the information that sexual
orientation should not matter
c. Requesting the information regarding the different types of groups
and discussing the choices with the client
d. Informing the person requesting the information that the client must
be consulted before that information can be released

Rationale: Information such as the sexual orientation of a client is confidential, and


the release of this information to a third party without the client’s permission
would constitute a violation of social worker-client confidentiality.

127. During an assessment for a parenting class, the parents of a 4 year old
express concern that the child may not be meeting developmental
milestones. The parents are concerned because the child has an imaginary
friend. The social worker reassures the parents that this is appropriate
behavior. The social worker interprets the behavior based on:
a. Sensorimotor intelligence
b. Preoperational thought
c. Concrete operational thought
d. Formal operational thought

Rationale: All four of the answers are Piaget’s developmental stages. (A) is the
first stage in which he applies to children birth to two years of age. (C) is the stage
from 7 to 11 years of age. (D) is the stage from 11 to adulthood. (B) is correct
because it applies to children from 2 to 7. Children in this stage of development
exhibit egocentric though and language.

128. A client tells a social worker that he has recently participated in a


violence criminal act and plans to repeat the offense. The social worker
should:
a. Increase sessions with the client to monitor the situation more closely
b. Maintain client confidentiality
c. Obtain a written contract that the client will not act on thoughts
d. Report the intent according to jurisdiction law

Rationale: Social workers must report information that may prevent imminent,
foreseeable harm to a client or other identifiable persons. The correct answer is D.
A, B, and C would not be appropriate as both the law and code of social work
ethics require this information to be reported.

129. A social worker supervisor is developing learning objectives with a


supervisee. The supervisee identifies areas of interest. Before completing a
training and staff development plan for the supervisee, the supervisor should
NEXT:
a. Consider the supervisor’s personal interest areas
b. Identify the specific competency requirements of the position
c. Obtain a list of qualified trainers
d. Develop a method for documenting training

Rationale: Identifying the specific competency requirement for the position will
enable the supervisor to develop the learning objectives specific to the job
description, therefore B is correct.

130. A social worker is interviewing a 15 year old client at a family


counseling center. The client reports having taken a whole bottle of
acetaminophen in a suicide attempt. The parents cannot be reached by the
social worker. The social worker should FIRST:
a. Asses the client’s mental status
b. Refer the client to an adolescent suicide prevention program
c. Obtain the client’s consent to call 911
d. Call 911

Rationale: The social worker is aware of the client’s intent of self-harm and the
primary duty becomes ensuring the client’s safety. Option D is correct because it is
an action that will immediately obtain help for the client. A and B do not address
the client’s immediate safety. Option C is incorrect because the social worker does
not need the client’s consent to obtain help.

131. A social worker is treating a client for depression. The client gives a
family history of multiple problems. To understand the relationship of the
family history to the client’s problems, the social worker should us a(n):
a. Ecomap
b. Genogram
c. Depression inventory
d. Sociogram

Rationale: Requires test-taker to recall the function of a genogram (B), particularly


as it relates to family history. Ecomaps (A) target present structures and
relationships, sociograms (D) are designed to map group members’ feelings about
each other, a depression inventory (C) would be of little use in understanding the
relationship of a client’s family history to current problems.

132. Which of the following activities MOST accurately measure client


progress during social work treatment?
a. Comparing the client’s current level of functioning in relation to
original treatment goals
b. Recording the decline in anxiety as reported by the client regarding
the presenting problem
c. Assessing the client’s ability to discuss painful difficulties more
openly than in initial sessions
d. Recording frequency of crisis calls and the client’s expressed need for
emergency sessions

Rationale: Client’s current functioning in relation to the original goals is a valid


measurement of progress, A is correct. B would mean the client was being treated
for anxiety, which is not indicated in the question. The same is true for C, there is
no evidence that the client has difficulty discussing painful issues. D is no
indicated because there is no evidence of crisis calls or an expressed need for
emergency sessions.

133. The social worker on a multidisciplinary team in a substance abuse


treatment program learns that the psychiatrist is recommending a client be
placed on antidepressant medication for depression. The client’s scheduled
discharge date is one week later and a halfway house placement has been
arranged. The hallway house will not accept clients on antidepressants. In a
discussion with the social worker, the client denies symptoms of depression
before the beginning of the substance abuse. The social worker should
FIRST:
a. Advise the client of the right to refuse the medication
b. Persuade the facility to accept the client on a trial basis
c. Discuss the referral problem with the psychiatrist and team
d. Request a conference with the halfway house administrator

Rationale: Two factors suggest that the use of an antidepressant may not be in the
best interest of the client: 1) the impending discharge plan would no longer be
viable and 2) the client reports no symptoms of depression before the substance
abuse problem began. In a multidisciplinary team, all members should be willing
to reconsider practice decisions based on the information provided by other team
members. In this case, the initial action should be to ask for a review of the
situation by the psychiatrist and other team members (C). Before the social worker
should ask another agency to make an exception (B) and (D), or encourage a client
to refuse the treatment of another team member (A), the referral issue should be
discussed directly among the team members.

134. A social worker is conducting intake interviews for applicants seeking


entrance into a subsidized housing program. The social worker needs to
FIRST determine an applicant’s:
a. Understanding of the program’s limitations
b. Knowledge of the program’s rules
c. Desire for improving personal circumstances
d. Ability to meet eligibility requirements

Rationale: Entrance to subsidized housing program is possible only if the client


meets the eligibility requirements. Thus it is only after the client’s eligibility is
determined (D) that the client’s motivation for improving his or her life
circumstances (C), understanding the program’s limitations (A), or familiarization
with the program rules (B) would be factors.
135. A nursing home social worker is responsible for assisting clients in
maintaining relationships with family. The social worker learns that a
resident’s family member recently disciplined a child with spanking that left
red marks. The social worker MUST:
a. Focus on the fact that the primary client is the nursing home resident
b. Speak discreetly to the family member about the situation
c. Make a referral for the family member to parenting classes
d. Make a report of suspected child abuse to the appropriate agency

Rationale: D is correct. Legal requirements for reporting suspected child abuse


may differ in various jurisdictions, it is generally legally and ethically required that
professionals report suspected abuse to appropriate authorities. Even though the
child may not be the primary client (A) of the worker, reporting is nevertheless
necessary. Other efforts to address the situation, such as speaking to a family
member (B) or referring to parenting classes (C), are not substitute for reporting
suspected child abuse.

136. The supervising social worker attends a client group facilitated by two
staff social workers. The supervisor is concerned that the topic being
pursued in the group session is a diversion of the group’s energy and a waste
of time. The social worker supervisor should:
a. Point out the concern to the group and redirect the session
b. Redirect the group and discuss the rationale with the social workers
later
c. Allow the group to continue and explore the concern afterwards
d. Encourage group members to identify the off task behavior
Rationale: Supervisor’s interference could affect the group dynamics and may
undermine the social workers’ authority within the group. Option A, B, and D
would all be inappropriate as they would undermine the therapeutic relationship
established by the worker and group members, therefore C is correct.

137. A community mental health social worker has a client who has
ignored four written requests regarding payment for services rendered. The
social worker plans to inform the client of the decision to proceed with
termination for nonpayment at the next regularly scheduled session. Before
proceeding with termination, the social worker should FIRST:
a. Arrange for a referral to another provider should the client want to
continue treatment
b. Examine the temporary option of bartering as a means of payment
c. Make certain that the client fully understands the details of the
payment for service agreement
d. Explore the possibility that nonpayment may represent an unconscious
resistance to authority

Rationale: Addresses ethical issues around termination. Although termination may


be the most appropriate action for the social worker to take in this situation, before
doing so the social worker must be sure that the client understands the reasons why
termination might be pursued (C). A referral may be appropriate (A) but it is not
the social worker’s FIRST action, nor should bartering be offered at this point (B).
Exploration of the possibility of unconscious resistance does not address the
immediate problem of nonpayment (D) and there is nothing in the stem to indicate
that the client has demonstrated any other behaviors that would warrant further
exploration.
138. Social workers in an inpatient psychiatric setting can BEST contribute
to the comprehensive treatment of a client by:
a. Establishing a relationship with the client’s doctor
b. Chairing meetings with the client’s family and with the treatment
team
c. Making sure all treatment team members attend staff meetings
d. Clarifying the social work role in the treatment process

Rationale: Ethically, social workers who are members of an interdisciplinary team


should understand the role of the treatment team as well as their own role. This
understanding should be conveyed to the client in order for the client to understand
the purpose and scope of the client-social worker relationship (D).

139. A social work student placed at a subsidized housing complex


organizes a community resource fair. The student advertises with public
service announcements, goes door to door explaining the fair, and has all
involved agencies place flyers in their service areas. This is an example of
community:
a. Advocacy
b. Outreach
c. Action
d. Involvement

Rationale: Advocacy (A) is the act of representing the community – not providing
information. Communication (C) involves change within the community to meet
goals. Community involvement is being generated by the social work student’s
activities but is not community involvement at this point (D). Option B is correct
answer. Bringing resources, services, and information about agencies to the public
is defined as outreach.

140. A social worker recently sold a car to an individual. The social worker
later discovers that the individual is the spouse of a current client. This is an
example of:
a. Countertransference
b. Boundary crossing
c. Communication breakdown
d. Boundary violation

Rationale: A boundary crossing is usually unintentional, occurs in isolation, and


may be discussed within the working therapeutic frame. By contrast, boundary
violations are usually intentional, repetitive, and are not usually discussed in the
therapeutic frame, so B is the correct response.

141. Since infancy, a child has experienced significant inconsistencies


from multiple caregivers. When the child begins school, a social worker is
MOST likely to notice that the child experiences:
a. Egocentric thoughts and feelings
b. Difficulty trusting adults
c. Preoccupation with peer relationships
d. Academic failure in math and science

Rationale: Unscored question. According to Erikson’s psychosocial theory of


development, a child will have difficulty in trusting adults (B) if there have been
significant inconsistencies from multiple caregivers. Option A, C, and D are not
necessarily results of a child experiencing inconsistencies from multiple
caregivers.

142. A social worker is counseling a client who has tested positive for
HIV. The client admits to not informing sexual partners of the diagnosis.
The social worker should FIRST:
a. Encourage the client to inform sexual partners
b. Inform the client’s partners of the positive test
c. Contact the local public health authority to discuss the issue
d. Take this as an opportunity to discuss AIDS prevention activities

Rationale: The social worker should first attempt to empower the client to inform
sexual partners of the HIV positive diagnosis (A). B and C may be appropriate in
some jurisdictions but not in others. Although a discussion of AIDS prevention
activities, D, would be appropriate at some point, it is not the FIRST thing a social
worker would do in this situation.

143. A social worker is unavoidably late in meeting with a client for a


scheduled appointment. The client is angry and begins to speak loudly and
shakes a finger in the social workers’ face. The social worker responds to
this behavior by saying ‘I’m glad that you are able to express your anger
about my being late”. According to communication theory, what technique
is the social worker using?
a. Positioning
b. Restraint from change
c. Reframing
d. Paradox

Rationale: Some people have answered this with question (D); the answer,
however, is (C). Paradoxical statements attempt to create double binds and in this
instance there is no double bind. Reframing is correct because the social worker is
restating the client’s statement in a more helpful manner and/or from a different
perspective.

144. A social worker employed at a women’s center assesses a female who


has bruises on her body. According to the client’s medical record, she has a
history of being assaulted by her husband. The client appears anxious and
vehemently denies being hit, claiming she accidentally fell down the stairs.
The client’s sense of powerlessness and denial is an indicator of:
a. Psychological abuse
b. Physical neglect
c. Mistrust of the system
d. An anxiety disorder

Rationale: Because the question indicates the woman’s sense of powerlessness and
denial, psychological abuse (A) is the correct answer. Psychological abuse
frequently accompanies physical abuse. There is not enough information in the
question to substantiate options C or D as a correct answer, and option B is already
suspected based on the physical evidence and history of abuse.

145. Practicing in a jurisdiction that grants social workers privileged


communication status means that the social workers can:
a. Share confidential information with other agencies
b. Refuse to accept a subpoena for a client’s records
c. Contest a request for confidential information
d. Ignore a client’s request for recorded information

Rationale: Client confidentiality is a client’s right and a social worker’s obligation


in jurisdictions where privileged communication statutes exist. In these
jurisdictions, social workers can legally withhold confidential client information
and contest request for such information (C). A social worker should never share
confidential information, (A), without the client’s consent. There is no choice but
to accept a subpoena, (B), although accepting it the social worker does not have to
immediately give up confidential information, but rather can legally seek and
‘quash’ the subpoena and not give the information. According to ethics and federal
law, clients have a right to see and have a copy of their records (D).

146. A client requests to use email to communicate with the social worker
while the client is out of town for several months. The social worker should
FIRST:
a. Offer the client a home telephone number for weekly check in
sessions
b. Inform the client about the limitations and risks of electronic
communication
c. Provide a list of social workers in the area where the client will be
staying
d. Transfer the client to a social worker with experience in long distance
counseling
Rationale: B is correct. Discussing the limitations and risks of electronic
communication provides the client with the opportunity to exercise informed
consent to use or not use the method. Depending on the discussion between the
client and social worker of the pros and cons of using email, the social worker may
offer a home telephone number (A), provide a list of social workers in the area
where the client will be (C), or transfer the client to someone with experience in
long-distance treatment (D), but none of these things should be done without
FIRST discussing the benefits and limitations of the client’s suggestions.

147. A social worker interviews a client at an inpatient mental health


facility. Although the client has been at the facility for six days, indicators of
homicidal and suicidal ideation continue. The client tells the social worker
of plans to leave the facility even though discharge has not been scheduled.
The social worker should NEXT:
a. Initiate an involuntary commitment process
b. Inform the client’s family members of the client’s plans to leave
c. Discuss with the client the likely consequences of this decision
d. Inform the staff of the client’s plan

Rationale: In this situation, the NEXT thing the social worker should do is talk
with the client, since the client is disclosing plans to leave the facility, therefore
(C) is correct. A, B, and D would all be things the social worker may do, but first
the client needs to be aware of the consequences of his or her actions.
148. A social worker is working with a client to create an effective
intervention plan. Of the many factors that influence the client’s success in
treatment, the social worker should FIRST discuss the client’s:
a. Expectations of the social worker’s role
b. Family background issues
c. Methods of problem solving used
d. Willingness to attend at least five sessions

Rationale: A is the correct answer. B, C, and D may all potentially be topics of


discussion in the beginning stages of treatment, but the FIRST thing the social
worker should do is to determine what the client needs and expects from the social
worker. This will assist the client and the social worker as they move forward
through the therapeutic process.

149. A high school teacher holds unrealistically high expectations for a


student who is Asian American. The school social worker BEST explains
the teacher’s expectations as a:
a. Basic ethnic stereotype
b. Discriminatory attitude
c. Motivational technique
d. Personal teaching style

Rationale: Question addresses the impact that stereotyping can have on behavior.
The teacher’s behavior is BEST explained as the result of an ethnic stereotype (A).
The question asks for an identification of the teacher’s expectations, so both C and
D are incorrect because they describe actions (motivational technique, teaching
style). Option B describes an attitude that would more frequently be associated
with expectations of poor performance.

150. A social worker is developing policies that will support service


delivery to persons who are disadvantaged. The FIRST action the social
worker should take is to:
a. Ensure the policies demonstrate a commitment to social justice
b. Review current professional research related to the issues involved
c. Identify traditions that will be supportive of the service delivery
d. Conduct an analysis of problems experienced by the population

Rationale: D is the correct answer. While the social worker may ensure policies
committed to social justice (A), review current related research (B), and identify
traditions that will support the service (C), none of these actions would be
conducted before an analysis of the problems experienced by the population.
Therefore, the FIRST thing that the social worker must do is to find out what the
problems are.

151. A hospital social worker meets with a family after a member is


critically injured and is in a coma. They family informs the social worker
that the patient has a same sex life partner, and they do not want this person
involved in the patient’s case. The life partner later arrives and asks the
social worker about the patient. The social worker should FIRST:
a. Refer the partner to the family for information
b. Inform the partner of the prognosis given by the doctor
c. Determine who can legally make health care decisions
d. Discuss the issue of medical decision making with the family

Rationale: Item asks about the FIRST step that the social worker should take in this
situation. Before discussing confidential information (B) or referring the partner to
the family for information (A) the social worker must determine who has been
authorized to receive the information (C). The social worker would also not discuss
the issue of medical decision-making with the family since it has not been
determined who has the authority to make those decisions (D).

152. A social worker assesses a client who has recently had a job loss and
is seeking job placement counseling. The social worker concludes that the
client is coping very well, drawing on internal qualities. In psychodynamic
theory, these qualities are referred to as:
a. Ego strengths
b. Internal balances
c. Genetic predispositions
d. Adaptations

Rationale: A is the correct answer. Ego strengths are the degree of psychic energy
available to the individual for problem solving, resolving internal conflicts, and
defending against mental and environmental distress. B, C, and D are possible
answers, but do not exist in psychodynamic theory.

153. A child protection social worker and a client are working toward the
return of the client’s children to the home. The client and the social worker
mutually agree on a plan. The client states a desire for the children’s return
but does not follow through with the plan. The social worker should FIRST:
a. Report the impression to the judge and request a court order
b. Determine if the client wishes to renegotiate the assignments
c. Assess the client’s understanding of the tasks’ relevance to the goals
d. Break the tasks into smaller steps so the client will not be
overwhelmed

Rationale: The social worker must be assured that the client understands why it is
important to follow the tasks assigned and how they relate to demonstrating that
the client is ready and prepared to receive the children back into the home,
therefore C is correct. (A) may be done if the social worker comes to the
conclusion that the parent is incapable of meeting or understanding the treatment
plan, which may put the children in danger or abuse or neglect. (B) would only be
an option in the beginning stages of treatment planning, if it is determined that
some of the assignments are not feasible for the parent. (D) may be done but only
after it is confirmed that the client understands the purpose of the tasks assigned.

154. During an initial session, a Native American/First Nations client


makes only brief eye contact and speaks softly with little apparent affect.
The social worker MOST likely can attribute this behavior to:
a. The presence of depression
b. Mistrust of the social worker
c. A culturally based expression of respect
d. Symptoms of paranoid schizophrenia

Rationale: Requires test-taker to apply knowledge of the ways in which culturally-


based behaviors may surface in the social worker – client relationship. It is too
early in the session to attribute this behavior to depression (A), paranoid
schizophrenia (D), or lack of trust (B).

155. During an initial therapy session with a client, a social worker realizes
that the client recently ended a romantic relationship with another client.
Neither client knows that the other is receiving services from the social
worker. The social worker is unsure if providing therapy to both clients is
ethical. After the social worker reviews the ethical standards, what else
should the social worker do to resolve the issue?
a. Ask the opinion of a trusted family member
b. Discuss the situation with the clients involved
c. Consult with knowledgeable social work colleagues
d. Continue to meet with the clients

Rationale: C is the correct answer. Consultation with colleagues can be very


beneficial to social workers struggling with possible ethical dilemmas. Asking the
opinion of a trusted family member (A) would violate confidentiality. Discussing
the situation with the clients involved (B) would not be the best way to resolve the
issue, as the clients would not be objective and each may not know the other is
receiving treatment. Continuing to meet with the clients (D) would not be the best
answer because the social worker is unsure of the ethical issues surrounding this
situation and until those are resolved, the social worker should not meet with either
one of the clients.

156. A social worker in a domestic violence center receives permission to


provide a pilot program for treatment of batterers. A review of the literature
indicates that a psychoeducational group is the most appropriate approach.
What is the NEXT step for the social worker to take?
a. Create a plan to measure the effectiveness of the group
b. Add individual therapy to the group treatment model
c. Train additional workers to add new groups for batterers
d. Educate local judges about the group’s existence

Rationale: A is the correct answer. Program planning always includes a way to


evaluate the program i.e., measures its effectiveness. It is too soon to add
individual therapy (B), add a new group for batterers (C), or educate judges about
the group (D) without knowing how the group treatment program will work. While
all three alternative answers, B, C, and D, might be possible, the group would need
to be functioning long enough to measure its effectiveness before the social worker
would initiate any of those choices.

157. A social worker has been assisting a client for several months. The
client fails to make payments for the services provided even though the
client was made aware of the agency’s fees and payment policy. The social
worker is considering discontinuing services to the client. What should the
social worker do?
a. Notify the client that services will be terminated after the next session
b. Remind the client of the payment agreement to ensure understanding
c. Terminate services immediately until the account is paid by the client
d. Locate financial resources to assist the client in paying for services

Rationale: The social worker must do what he or she can to make sure that the
client understands the payment expectations (B), before terminating services
prematurely (C), which may be unethical and could put the client at risk. Assisting
the client in locating financial resources (D) and notifying the client that services
will be terminated after the next session (A), may be something the social worker
would do but only after the social worker is assured that the client understands the
payment contract.

158. A social worker is concerned that a therapeutic group is not


developing cohesion. This prevents the opportunity for more intense work.
The social worker decides to use several trust building exercises with the
group. What should the social worker do FIRST?
a. Identify the group member most willing to take risks
b. Establish rules about taking risks in the group
c. Talk with the group about any concerns with taking risks
d. Give group members an opportunity to rotate out of the group

Rationale: Correct answer is C. Identifying the specific group members willing to


take risks (A) will not encourage group cohesion and will isolate other members of
the group. Establishing rules about taking risks in the group (B) would be
appropriate to do, but would not be done first. Giving the group members an
opportunity to rotate out of the group (D) is not an option. Possible reasons for
rotating out should be established in the beginning of the group process and should
not be revisited throughout each phase of the group treatment.

159. A social worker receives a report of alleged child abuse about a family
with prior substantiated allegations of abuse of their child. The social worker
interviews the child’s parents, who are both age 17. The child, who is 4
years old, reports physical and emotional abuse by the parents. Which factor
should carry the MOST weight in the social worker’s assessment of abuse?
a. The unmarried status of the child’s parents
b. The age of the child
c. The history of previously reported abuse
d. The underage status of the child’s parents

Rationale: History of previous abuse (C) is the factor MOST strongly correlated
factor in the assessment of current abuse. The parents’ marital status (A), while a
possible stressor, is not identified in the stem; the age of the child (B) does not
relate to an assessment of current abuse; and the ages of the parents (D), while a
possible stressors, does not present the strongest correlation to an assessment of
current abuse.

160. A 16 year old meets with a social worker because of his recent
reluctance to attend school. His previous attendance has not been a problem
and he is considered a good student. In this situation, the social worker
should FIRST:
a. Initiate a referral for a psychological evaluation
b. Explore possible precipitating factors
c. Consult parents about change in the home
d. Refer the student for drug and alcohol screening

Rationale: Unscored question. Teenagers often experience periods of reluctance to


attend school – even when they have not previously displayed attendance
problems. While poor attendance might be an initial symptom of individual
psychological (A) and/or substance abuse (D) problems – or evolving issues in
family life (C) – to move prematurely to assess these matters would represent
overreaction and might also obscure other sources of the problem. Instead, the
worker should start the process by exploring with the client his view of factors that
might be contributing to the attendance problems (B) as a means of both showing
respect for the client’s insights and addressing the problem at the most basic level.

161. An adolescent has been referred to the school social worker because
of hostility and fighting with peers. To establish a therapeutic relationship
with this client, the social worker should FIRST:
a. Discuss confidentiality limitations with the student
b. Advocate with the school administrators to change the student’s
school program
c. Validate the student’s hostile feelings and explore reasons for fighting
d. Reduce the student’s tension by discussing school activities

Rationale: Unscored question. Discussing the confidentiality limitations (A) is one


of the first things that social worker should do with clients as this is prerequisite to
the client sharing information with informed consent. In this situation, the social
worker would next reduce the student’s tension (D), then validate the student’s
hostile feelings and explore reasons for fighting (C). Later, there may be a need for
the social worker to advocate with the school administration to change the
student’s school program (B).

162. A teacher of third graders refers a child to the school social worker.
The teacher complains that the child is easily distracted, calls out in the
classroom, does not finish assigned work, and displays occasional temper
tantrums. Development history obtained by the social worker reveals that the
child has always been impulsive and has had difficulty following directions.
According to the DSM-IV-TR classification, the diagnosis will MOST likely
be:
a. Oppositional defiant disorder
b. Conduct disorder
c. Attention deficit hyperactivity disorder, predominantly hyper active
type
d. Pervasive developmental disorder

Rationale: Unscored question. The child’s behavior describes the characteristics of


attention-deficit hyperactivity disorder (C) as described in the DSM IV TR.
Oppositional defiant disorder (A) is characterized by defiant behavior, refusal to
follow rules, blaming others for mistakes, anger and hateful behavior; conduct
disorder (B) is characterized by aggression toward people and animals, serious
violations of rules, deceitfulness, lying or stealing; and destruction of property.
Pervasive development disorder (D) is characterized by difficulty in
communicating as with autism.

163. Children who suffer from chronic neglect will MOST likely exhibit:
a. Sexual acting out
b. Hypervigiliance
c. Compulsivity
d. Decreased intellectual ability

Rationale: Recall item that tests knowledge of effects of neglect.


164. A social worker is evaluating the level of functioning of an elderly
client admitted to the hospital for severe malnutrition. During the interview,
the client displays anxiety, confusion, disorientation, which is not usual for
this individual. The client’s emotional state is MOST likely due to:
a. Neglect
b. Dysthymia
c. Korsikoff’s disease
d. Delirium

Rationale: Question requires test-taker to match symptoms to a diagnosis. The


symptoms identified in the stem (anxiety, confusion, and disorientation), coupled
with the fact that the symptoms are described as being ‘not usual’ for the client,
make delirium the MOST likely explanation for the client’s state. Korsikoff’s
disease (C) is an amnesia related to prolonged alcohol abuse; neglect (A) and
dysthymia (B) do not generally include the symptoms described here.

165. Several clients who have physical disabilities attend a social worker’s
life skills class. The clients express frustrations about voting in elections
because they have heard polling places are not handicapped accessible. After
determining that the clients’ concerns are valid, the social worker should:
a. Obtain legal advice relating to the clients’ concerns
b. Obtain absentee ballots for the clients to complete during class
c. Respect the clients’ rights to refrain from voting in the election
d. Empower the clients to educate local officials on the needs of voters
with disabilities
Rationale: The social worker’s task in this scenario is to both validate the concerns
of the clients while assisting them in taking action that will net long-term results.
Empowering the group to bring their message to local officials (D) is the approach
that will best meet these goals. Obtaining absentee ballots (B) does not generate
any permanent changes; respecting the clients’ right to not vote (C) is a passive
approach that does not validate the clients’ frustrations; obtaining legal service (A)
may be appropriate later, but not until the clients have attempted to have their
concerns addressed through discussions with local officials.

166. A new social worker provides case management services for a client
and suspects an eating disorder. What should the social worker do FIRST to
locate appropriate services?
a. Consult with colleagues who are knowledgably about resources
b. Read about eating disorder therapies in a professional journal
c. Refer the client to a national eating disorder website
d. Refer the client to a comprehensive mental health center

Rationale: Quesiton addresses the test-taker’s knowledge of the use of consultative


approaches. Reading about eating disorder therapies (B) may be informative, but is
not the FIRST action that should be taken to locate appropriate services.
Suggesting that the client visit a website on eating disorders (C) does not address
the need for services, and there is no firm basis for referral to a mental health
center (D). Colleagues who know about resources (A) are the social worker’s
FIRST resource in this situation.
167. A social worker responds to an advertisement in the paper for a
puppy, and learns that it is owned by a former client. The former client
recognizes the social worker and offers the puppy at half price. The social
worker’s BEST response is to:
a. Accept the offer since it is a former client who is involved
b. Make the purchase at the advertised price
c. Let the client decide how much to ask for the puppy
d. Have a colleague negotiate the transaction

Rationale: Basic boundary issue that requires an answer rooted solidly in social
work ethics. Purchasing the puppy at the advertised price (B) does not compromise
the past social worker-client relationship. Accepting the offer for a price reduction
(A) is unethical. Compromising on a price (C) or getting a colleague involved in
the transaction (D) both amount to an unacceptable twisting of ethical standards.

168. A social worker at an alcohol dependency treatment center is working


with a 53 year old male client who claims he has remained completely sober
for two weeks. The client is concerned because he recently was not able to
perform sexually for the first time in his life. In formulating a response, the
social worker should FIRST consider that sexual dysfunction:
a. May occur during withdrawal and the early stages of recovery
b. Is most likely due to the client’s age and history of substance abuse
c. Shows clear evidence of relapse and recent alcohol consumption
d. Could be evidence of the early stages of Korsikoff’s disease

Rationale: Requires test-takers to have knowledge of the possible physical side-


effects of early recovery. Sexual dysfunction (A) is one such fairly common side-
effects. While client age and substance abuse history may be a factor (B), this
consideration should not be considered the most likely reason for the problems.
There is no evidence in the stem of relapse (C) and the symptoms described are not
consistent with Korsikoff’s disease (D).

169. A social worker with a home health agency assesses a client for
possible dementia. The client lives alone and receives home health services.
The nursing aide states that the client’s cognitive abilities have changed
significantly in the past several days. In addition to being confused and
disoriented, the client is unsteady when walking. What should the social
worker do FIRST?
a. Refer the client for a battery of psychological tests
b. Arrange a medical exam for the client as soon as possible
c. Refer the client for adult day care services
d. Discuss possible care placement with the client

Rationale: The FIRST action the social worker should take is to investigate
whether the symptoms described in this question have a neurological or organic
basis (B). Ignoring the physical symptoms in favor of psychological testing (A),
puts the client at risk, as does making a referral for adult daycare (C), and
discussing placement options (D). The recent onset of these symptoms requires
immediate action to rule out a medical condition.

170. A social worker is working with a client who experiences ongoing


physical abuse from her partner. The social worker has reviewed and
complied with reporting requirements. The social worker explores the
situation further and finds that the client makes excuses for the partner. The
client also refuses to go to a shelter. What should be the social worker’s
NEXT step?
a. Ask the client to have her partner come to their next meeting
b. Obtain more information about the client’s relationship with the
partner
c. Assist the client in creating a safety plan in case it is needed
d. Help the client identify her coping skills and strengths

Rationale: Safety plans (C) are an essential component in interventions that involve
domestic violence. Asking that the client have her partner come to the next
meeting (A) may actually escalate the potential for violence, while helping the
client identify coping skills and strengths (D) does not address the immediate
needs of the client. Obtaining further information (B) is not necessary in order for
the social worker to take action in this situation.

You might also like